Вы находитесь на странице: 1из 33

https://t.me/pdf4exams www.pdf4exams.

in

KSG: MEMBER’S COPY


TEST – 4
(EXPLANATION & SOURCE)

1. Constitution of India is said to be (b) Historical factors, e.g., the influence of


Synthesis of both rigidity and flexibility the Government of India Act of 1935,
because of what features? which was bulky.
1. Some provisions can be amended (c) Single Constitution for both the Centre
by special majority. and the states
2. Some provisions can be amended (d) Dominance of legal luminaries in the
by simple majority. Constituent Assembly. The Constitution
3. Some provisons can‘t be amended contains not only the fundamental
at all. principles of governance but also
4. Some provisions can be amended detailed administrative provisions.
by ordinance by the president. (e) Diverse culture
Select the correct answer using the Source:LaxmikanthVth edition, Chapter 3,
code given below: page no 3.2
(a) 1 and 2 only
(b) 2, 3 and 4 only 3. Consider the following pairs:
(c) 1, 2 and 4 only Articles Provisions
(d) 1, 2, 3 and 4 1. Article 243 Panchayats
2. Article 343 Schedule tribes
Explanation: 3. Article 40 Living wage for
Some provisions are amended by simple workers
majority and some are by special majority. Which of the pairs given above is/are
No provisions are there which can‘t be correctly matched?
amended and no provisions can be amended (a) 1 only
through ordinance of the president. (b) 1 and 2 only
Source:LaxmikanthVth edition, Chapter 3, (c) 3 only
page no 3.3 (d) 1, 2 and 3
Explanation:
2. What are the factors which contributed Pairs 2 and 3 are incorrect
to the Elephantine size of the Indian
Article 343: Official language of the union
Constitution?
Article 40: Organisation of village panchayats
1. Geographical factors
Source:LaxmikanthVth edition, Chapter 3,
2. Diverse culture
page no 3.12, 3.13 &3.14
3. Historical influence
4. Dominance of legal luminaries in
4. Which of the following provisions were
the constituent assembly
borrowed from the Government of India
5. Single constitution Act 1935?
Select the correct answer using the 1. Parliamentary privileges
code given below:
2. Emergency provisions
(a) 2, 3 and 5 only
3. Public service commissions
(b) 1, 2, 3 and 5 only
4. Bicameralism
(c) 1, 2 and 4 only
5. Federal scheme
(d) 1, 2, 3, 4 and 5
Select the correct answer using the
Explanation: code given below:
Four factors have contributed to the (a) 2, 3 and 5 only
elephantine size of our Constitution.
(b) 2, 3 and 4 only
They are:
(c) 1, 2 and 5 only
(a) Geographical factors, that is, the
(d) 1, 2, 3, 4 and 5
vastness of the country and its diversity.
Explanation:
Parliamentary Privileges and Bicameralism
were borrowed from British Constitution.
[TEST- 4 EXPLANATION] [KSG: 011-45552607, 47640303, 0141-4052441, 0755-4077441, 09386337412] Page 1

Googlw it: pdf4exams


https://t.me/pdf4exams www.pdf4exams.in

KSG: MEMBER’S COPY


Provisions borrowed from Government of Explanation:
India Act 1935: Federal Scheme, Office of Statement 1 is incorrect
Governor, Judiciary, Public Service The Indian brand of socialism is a
Commissions, Emergency provisions and ‗democratic socialism‘ and not a
administrative details. ‗communistic socialism‘ (also known as ‗state
Source:LaxmikanthVth edition, Chapter no socialism‘) which involves the nationalisation
3, page no 3.11 of all means of production and distribution
and the abolition of private property.
5. Consider the following statements Democratic socialism, on the other hand,
about the Preamble of the constitution: holds faith in a ‗mixed economy‘ where both
1. It is the Preface to the constitution. public and private sectors co-exist side by
2. It is based on the Objective side. As the Supreme Court says,
Resolution. ‗Democratic socialism aims to end poverty,
ignorance, disease and inequality of
3. It cannot be amended, as it is not
opportunity.
a part of the constitution.
Statement 2 is correct
Which of the statements given above
is/are correct? Indian socialism is a blend of Marxism and
Gandhism, leaning heavily towards
(a) 1 only
Gandhian socialism.
(b) 1 and 2 only
Source:LaxmikanthVth edition, Chapter 4,
(c) 2 and 3 only page no 4.3
(d) 1, 2 and 3
Explanation: 7. Match the following:
Statement 1 is correct
Term Implication
The term ‗preamble‘ refers to the
introduction or preface to the Constitution. It (A) Distributive (1) Elimination of
contains the summary or essence of the justice glaring
Constitution. N APalkhivala, an eminent inequalities in
jurist and constitutional expert, called the wealth, income
Preamble as the ‗identity card of the andProperty.
Constitution.‘ (B) Social (2) Equal voice in
Statement 2 is correct Justice the government.
The Preamble to the Indian Constitution is (C) Economic (3) A combination of
based on the ‗Objectives Resolution‘, drafted Justice social justice
and moved by Pandit Nehru, and adopted by and economic
the Constituent Assembly. justice.
Statement 3 is incorrect (D) Political (4) Absence of
In 1976, the Preamble was amended (only Justice privilege to any
once till date) by the 42nd Constitutional particular
Amendment Act. group.
Source:LaxmikanthVth edition, Chapter 4,
page no 4.1 Select the correct answer using the
code given below
6. Consider the following statements with
respect to the Socialistic character of A B C D
the Indian state:
(a) 3 1 4 2
1. The Indian brand of socialism is a
(b) 1 4 2 3
‗State socialism‘.
(c) 3 1 2 4
2. Indian socialism is a blend of
Marxism and Gandhism. (d) 3 4 1 2
Which of the statements given above Explanation:
is/are correct? Social justice denotes the equal treatment of
(a) 1 only all citizens without any social distinction
based on caste, colour, race, religion, sex
(b) 2 only
and so on. It means absence of privileges
(c) Both 1 and 2 being extended to any particular section of
(d) Neither 1 nor 2 the society, and improvement in the

[TEST- 4 EXPLANATION] [KSG: 011-45552607, 47640303, 0141-4052441, 0755-4077441, 09386337412] Page 2

Googlw it: pdf4exams


https://t.me/pdf4exams www.pdf4exams.in

KSG: MEMBER’S COPY


conditions of backward classes (SCs, STs 9. Consider the following statements
and OBCs) and women. abouttheVoter Verifiable Paper Audit
Economic justice denotes the non- Trail (VVPAT):
discrimination between people on the basis 1. It is an independent verification
of economic factors. It involves the printer machine, attached to
elimination of glaring inequalities in wealth, electronic voting machines.
income and property. 2. The slip shows only the poll
A combination of social justice and economic symbol.
justice denotes what is known as 3. It is manufactured by Bharat
‗distributive justice‘. Electronics Limited.
Political justice implies that all citizens Which of the statements given above
should have equal political rights, equal is/are correct?
access to all political offices and equal voice (a) 1 only
in the government.
(b) 2 only
Source:LaxmikanthVth edition, Chapter 4,
(c) 1 and 3 only
page no 4.3
(d) 1, 2 and 3
Explanation:
8. Consider the following statements
about the Service Voters: Statement 1 is correct
1. It has been defined in the The Voter Verifiable Paper Audit Trail is a
constitution of India. method that provides feedback to voters. It is
an independent verification printer machine
2. These Voters comprise of the
and is attached to electronic voting
Central civil servants posted in
machines.
other Indian states.
Statement 2 is incorrect
3. They cast their vote via proxy voter
and postal ballot mode. The slip contains the poll symbol, candidate
serial number and name of the candidate.
Which of the statements given above
is/are correct? Statement 3 is correct
(a) 1 only It is manufactured by Bharat Electronics
Limited and Electronics Corporation of India
(b) 1 and 2 only
Limited.
(c) 3 only
Source: KSG Current Connect, April 2019,
(d) 2 and 3 only page no 11, 12
Explanation:
Statement 1 is incorrect 10. What are the provisions in the
These voters are defined according to the constitution of India that secure Civic
provisions of sub-section (8) of Section 20 of Equality?
Representation of People Act, 1950. 1. Article 15: Prohibition of
Statement 2 is incorrect discrimination on grounds of
Service Voters are: religion, race, caste, sex or place of
1. Those serving in the Armed Forces of the birth.
Union. 2. Article 39: Secures to men and
2. Those serving in a Force to which the women equal right to an adequate
Army Act 1950 applies (Assam rifles, means of livelihood and equal pay
CRPF, BSF, ITBP, SSB, NSG, GREF in for equal work.
BRO (Border Road organisation), CISF 3. Article 16: Equality of opportunity
etc. in matters of public employment.
3. Member of an Armed Police Force of a 4. Article 326:Elections to the
State, serving outside that state. 4. LokSabha and the state
Those employed under the Government assemblies to be on the basis of
of India, in a post outside India. adult suffrage.
Statement 3 is correct Which of the given pairs are
Tool for Service Voter to cast their vote: incorrectly matched?
Proxy Voter, Postal Ballot and Electronically (a) 1 and 4 only
transmitted Postal Ballot System (ETPBS). (b) 2 and 3 only
Source:KSG Current Connect, April 2019, (c) 2 and 4 only
page no 10
[TEST- 4 EXPLANATION] [KSG: 011-45552607, 47640303, 0141-4052441, 0755-4077441, 09386337412] Page 3

Googlw it: pdf4exams


https://t.me/pdf4exams www.pdf4exams.in

KSG: MEMBER’S COPY


(d) 1 and 2 only They are justiciable, allowing persons to
Explanation: move the courts for their enforcement, if and
Statement 2 and 4 are incorrect when they are violated.
Article 39 is for Economic equality and Statement 3 is incorrect
article 326 is for political equality. They are not sacrosanct or permanent. The
 The following provisions of the chapter Parliament can curtail or repeal them but
on Fundamental Rights ensure civic only by a constitutional amendment act and
equality: not by an ordinary act. Moreover, this can be
done without affecting the ‗basic structure‘ of
(a) Equality before the law (Article 14).
the Constitution.
(b) Prohibition of discrimination on
Source:LaxmikanthVth edition, Chapter 7,
grounds of religion, race, caste, sex
page 7.8
or place of birth (Article 15).
(c) Equality of opportunity in matters of
public employment (Article 16). 12. In which of the following cases it was
upheld that the Preamble is a part of
(d) Abolition of untouchability (Article
the Constitution of India?
17).
1. Berubari Union case
(e) Abolition of titles (Article 18).
2. KesavanandaBharti case
There are two provisions in the Constitution
that seek to achieve political equality: 3. LIC of India case
One, no person is to be declared ineligible for 4. Raj Narain case
inclusion in electoral rolls on grounds of Select the correct answer using the
religion, race, caste or sex (Article 325). code given below:
Two, elections to the LokSabha and the state (a) 2 and 4 only
assemblies to be on the basis of adult (b) 2 and 3 only
suffrage (Article 326). (c) 1, 3 and 4 only
The Directive Principles of State Policy (d) 1, 2 and 3 only
(Article 39) secures to men and women equal Explanation:
right to an adequate means of livelihood and
In the Berubari Union case (1960), the
equal pay for equal work.(Economic equality)
Supreme Court said that the Preamble
Source:LaxmikanthVth edition, Chapter 4, shows the generalpurposes behind the
page no 4.7 several provisions in the Constitution, and is
thus a key to the minds of the makers of the
11. Consider the following statements Constitution. Further, where the terms used
about Fundamental Rights: in any article are ambiguous or capable of
1. They are absolute in nature. more than one meaning, some assistance at
2. They are justiciable in nature. interpretation may be taken from the
objectives enshrined in the Preamble.
3. They are sacrosanct and
Despite this recognition of the significance of
permanent.
the Preamble, the Supreme Court specifically
Which of the statements given above opined that Preamble is not a part of the
is/are correct? Constitution.
(a) 1 only In the KesavanandaBharati case (1973), the
(b) 2 and 3 only Supreme Court rejected the earlier opinion
(c) 2 only and held that Preamble is a part of the
(d) 1, 2 and 3 Constitution. It observed that the Preamble
Explanation: is of extreme importance and the
Constitution should be read and interpreted
Statement 1 is incorrect in the light of the grand and noble vision
They are not absolute but qualified. The expressed in the Preamble.
state can impose reasonable restrictions on In the LIC of India case (1995) also, the
them. However, whether such restrictions Supreme Court again held that the Preamble
are rea-sonable or not is to be decided by the is an integral part of the Constitution.
courts. Thus, they strike a balance between
the rights of the individual and those of the Raj Narain case: It was the case to apply
society as a whole, between individual liberty basic structure doctrine. In this case, Court
and social control. had to examine the validity of the Thirty
Ninth Amendment Act by whichArt.329-A
Statement 2 is correct was inserted into the Constitution.
[TEST- 4 EXPLANATION] [KSG: 011-45552607, 47640303, 0141-4052441, 0755-4077441, 09386337412] Page 4

Googlw it: pdf4exams


https://t.me/pdf4exams www.pdf4exams.in

KSG: MEMBER’S COPY


Source:LaxmikanthVth edition, Chapter 4, 14. Consider the following statements:
page no 4.9 1. Union of India includes only
https://indiankanoon.org/doc/936707/ states.
2. Territory of India includes not only
13. Consider the following pairs: states but Union Territories also.
Devices of Procedure/Method Which of the statements given above
direct is/are correct?
democracy (a) 1 only
1. Referendum Procedure whereby a (b) 2 only
proposed legislation (c) Both 1 and 2
is referred to (d) Neither 1 nor 2
the electorate for Explanation:
settlement by their Both the statements are correct
direct votes. The ‗Territory of India‘ is a wider expression
2. Initiative Method of obtaining than the ‗Union of India‘ because the latter
the opinion of India‘s includes only states while the former
people on any includes not only the states but also union
issue of public territories and territories that may be
importance. acquired by the Government of India at any
future time. The states are the members of
3. Recall Method by means of
the federal system and share a distribution
which the voters can
of powers with the Centre.
remove
representative before The union territories and the acquired
the expiry of his territories, on the other hand, are directly
term, when he fails administered by the Central government.
to discharge his Source:LaxmikanthVth edition, Chapter 5,
duties properly. page no 5.2
4. Plebiscite Method by means of
which the people 15. Which of the following statements is
can propose a bill to correct?
the legislature for (a) Laws made for establishment or
enactment. admission and formation of new
states are to be considered as
Which of the pairs given above is/are
amendments of the constitution
correctly matched?
and requires a special majority.
(a) 1 and 2 only
(b) The power of parliament to
(b) 2 and 3 only diminish the areas of a state
(c) 3 and 4 only (under Article 3) include also the
(d) 1 and 3 only power to cede Indian Territory to a
Explanation: foreign country.
Referendum is a procedure whereby a (c) Settlement of a boundary
proposed legislation is referred to the dispute between India and
electorate for settlement by their direct votes. another country can be done by
Initiative is a method by means of which the executive action.
people can propose a bill to the legislature (d) None of the above
for enactment. Explanation:
Recall is a method by means of which the Statement 1 is incorrect
voters can remove a representative or an The Constitution (Article 4) itself declares
officer before the expiry of his term, when he that laws made for admission or
fails to discharge his duties properly. establishment of new states (under Article 2)
Plebiscite is a method of obtaining the and formation of new states and alteration of
opinion India‘s of people on any issue of areas,boundaries or names of existing states
public importance. It is generally used to (under Articles 3) are not to be considered as
solve the territorial disputes. amendmentsof the Constitution under Article
Source:LaxmikanthVth Edition, Chapter no 368. This means that such laws can be
4, page no 4.12

[TEST- 4 EXPLANATION] [KSG: 011-45552607, 47640303, 0141-4052441, 0755-4077441, 09386337412] Page 5

Googlw it: pdf4exams


https://t.me/pdf4exams www.pdf4exams.in

KSG: MEMBER’S COPY


passed by a simplemajority and by the 17. The Election Commission of India
ordinary legislative process. administers elections to the:
Statement 2 is incorrect 1. LokSabha
After the controversy of the Berubari union 2. RajyaSabha
(1960), the Supreme Court held that the 3. State Legislative Assemblies in
power of Parliament to diminish the area of a India
state (under Article 3) does not cover cession 4. The office of the President
of Indian Territory to a foreign country.
5. The office of Speaker of the
Hence, Indian Territory can be ceded to a
LokSabha
foreign state only by amending the
Constitution under Article 368. Select the correct answer using the
codes given below:
Consequently, the 9th Constitutional
Amendment Act (1960) was enacted to (a) 1, 2 and 3 only
transfer the said territory to Pakistan. (b) 1, 2, 3 and 4 only
Statement 3 is correct (c) 1 and 3 only
On the other hand, the Supreme Court in (d) All of the above
1969 ruled that, settlement of a boundary Explanation:
dispute between India and another country The Election Commission of India is an
does not require a constitutional autonomous constitutional authority
amendment. It can be done by executive responsible for administering Union and
action as it does not involve cession of Indian State election processes in India. The body
Territory to a foreign country. administers elections to the LokSabha,
Source:LaxmikanthVth edition, Chapter 5, RajyaSabha and State Legislative Assemblies
page no 5.3 &5.4 in India, and the offices of the President and
Vice President in the country.
16. Consider the following pairs: Source: KSG Current Connect, April 2019,
Region Integration by page no 13
1. Kashmir The Instrument of
Accession. 18. Consider the following statements
2. Hyderabad Means of about the Traditional Knowledge
Referendum. Digital Library (TKDL):
3. Junagarh Means of Police 1. It has been developed by NITI
action. Aayog in collaboration with the
Ministry of Health and family
Which of the pairs given above is/are
welfare.
correctly matched?
2. It seeks to prevent the granting of
(a) 1 only
patents for products developed
(b) 2 and 3 only utilizing traditional Knowledge
(c) 2 only with little inventive step.
(d) 1, 2 and 3 3. It is available in multiple
Explanation: languages.
The Indian Independence Act (1947) created Which of the statements given above
two independent and separate dominions of is/are correct?
India and Pakistan and gave three options to (a) 1 only
the princely states viz., joining India, joining (b) 2 only
Pakistan or remaining independent. Most of
(c) 2 and 3 only
the princely states situated within the
geographical boundaries joined India and the (d) 1, 2 and 3
remaining 3 (Hyderabad, Junagarh and Explanation:
Kashmir) refused to join India. Statement 1 is incorrect
However, in course of time, they were also CSIR jointly with Department of AYUSH (now
integrated with India—Hyderabad by means Ministry) developed the Traditional
of police action, Junagarh by means of Knowledge Digital Library (TKDL).
referendum and Kashmir by the Instrument It is a globally recognized proprietary
of Accession. database on Indian systems of medicine for
Source:LaxmikanthVth edition, Chapter 5, preventing bio-piracy and misappropriation
page no 5.6 of our traditional knowledge.

[TEST- 4 EXPLANATION] [KSG: 011-45552607, 47640303, 0141-4052441, 0755-4077441, 09386337412] Page 6

Googlw it: pdf4exams


https://t.me/pdf4exams www.pdf4exams.in

KSG: MEMBER’S COPY


Statement 2 is correct 20. Arrange the following events in correct
It seeks to prevent the granting of patents for chronological order:
products developed utilizing TK where there 1. Formation of state of Himachal
has been little, if any, inventive step. Pradesh
It intends to act as a bridge between 2. Formation of state of Gujrat
information recorded in ancient Sanskrit and 3. Formation of state of Arunachal
patent examiners (with its database Pradesh
containing information in a language and 4. Formation of state of Sikkim
format understandable to patent examiners).
Select the correct answer using the
It facilitates access to information not easily code given below
available to patent examiners, thereby
(a) 2-4-1-3
minimizing the possibility that patents could
be granted for ―inventions‖ involving only (b) 1-2-4-3
minor or insignificant modifications. (c) 2-1-4-3
Statement 3 is correct (d) 2-1-3-4
It is available in five languages: English, Explanation:
German, French, Japanese and Spanish. Gujrat was the 15th state of Indian Union
Source:KSG Current Connect, April 2019, (1960)
page no 7 Himachal Pradesh: 17th state (1971)
Sikkim: 22nd state (1974)
19. What were the factors identified by the Arunachal Pradesh: 24th state (1987)
Fazl Ali commission that can be taken Source:LaxmikanthVth edition, Chapter 5,
into account in any scheme of page no 5.11, 5.12
reorganisation of states?
1. Financial, Economic and 21. Whichof the following rights/provisions
Administrative. are denied to the aliens in the
2. Planning and promotion of social constitution of India?
welfare. 1. Article 15
3. Linguistic and cultural 2. Article 22
homogeneity. 3. Article 29
4. Preservation and the strengthening 4. Right to contest for the
of unity and security of the nation. membership of the state legislature
Select the correct answer using the Select the correct answer using the
code given below: code given below:
(a) 1 and 3 only (a) 1, 3 and 4 only
(b) 3 only
(b) 1 and 4 only
(c) 1, 2 and 4 only
(c) 2, 3 and 4 only
(d) 1, 2, 3 and 4
(d) 1, 2, 3 and 4 Explanation:
Explanation: The Constitution confers the following rights
Fazl Ali commission identified four major and privileges on the citizens of India (and
factors that can be taken into account in any denies the same to aliens):
scheme of reorganisation of states: 1. Right against discrimination on grounds
(a) Preservation and strengthening of the of religion, race, caste, sex or place of
unity and security of the country. birth (Article 15).
2. Right to equality of opportunity in the
(b) Linguistic and cultural homogeneity.
matter of public employment (Article 16).
(c) Financial, economic and administrative 3. Right to freedom of speech and
considerations. expression, assembly, association,
(d) Planning and promotion of the welfare of movement, residence and profession
the people in each state as well as of the (Article 19).
nation as a whole. 4. Cultural and educational rights (Articles
Source:LaxmikanthVth edition, Chapter 5, 29 and 30).
page no 5.8 5. Right to vote in elections to the
LokSabha and state legislative assembly.

[TEST- 4 EXPLANATION] [KSG: 011-45552607, 47640303, 0141-4052441, 0755-4077441, 09386337412] Page 7

Googlw it: pdf4exams


https://t.me/pdf4exams www.pdf4exams.in

KSG: MEMBER’S COPY


6. Right to contest for the membership of Residence) Act, 1957 and thereby
the Parliament and the state legislature. authorised the Government of India to
7. Eligibility to hold certain public offices, prescribe residential qualification only
that is, President of India, Vice- for appointment to non-Gazetted posts
President of India, judges of the in Andhra Pradesh, Himachal Pradesh,
Supreme Court and the high courts, Manipur and Tripura.
governor of states, attorney general of (3) The freedom of movement and residence
India and advocate general of states. (under Article 19) is subjected to the
Source:LaxmikanthVth Edition, Chapter 6, protection of interests of any schedule
page no 6.1 tribe. In other words, the right of
outsiders to enter, reside and settle in
tribal areas is restricted.
22. In India, all citizens irrespective of the
state in which they are born or reside Statement 3 is incorrect: Article 17 is
enjoy the same political and civil rights related to Abolition of untouchability.
of citizenship all over the country and Source:LaxmikanthVth edition, Chapter 7,
no discrimination is made between page no 7.6
them. However, this general rule of
absence of discrimination is subject to 23. The Article 12 of the constitution of
some exceptions. Which of the India has defined the term State for the
following articles in Indian constitution purpose of Part III of the constitution.
provide these exceptions? Which of the following
1. Article 15 organisation/institution comes under
2. Article 16 the definition of State?
3. Article 17 1. Municipalities and Local bodies.
4. Article 19 2. Central statistics Office.
Select the correct answer using the 3. ONGC and SAIL
code given below: 4. Improvement trusts
(a) 2 and 4 only 5. Life Insurance Corporation
(b) 1, 2 and 4 only 6. National commission for Schedule
(c) 1 and 3 only caste and schedule tribes.
(d) 1, 2, 3 and 4 Select the correct answer using the
Explanation: code given below:
Statements 1, 2 and 4 are correct (a) 1, 2, 4 and 6 only
The general rule of absence of discrimination (b) 2, 3 and 6 only
is subject to some exceptions, viz, (c) 1, 3, 4 and 5 only
(1) The Constitution (under Article 15) (d) 1, 2, 3, 4, 5 and 6
prohibits discrimination against any Explanation:
citizen on grounds of religion, race, The term ‗State‘ has been used in different
caste, sex or place of birth and not on provisions concerning the fundamental
the ground of residence. This means rights. Hence, Article 12 has defined the
that the state can provide special term for the purposes of Part III. According to
benefits or give preference to its it, the State includes the following:
residents in matters that do not come
(a) Government and Parliament of India,
within the purview of the rights given by
that is, executive and legislative organs
the Constitution to the Indian citizens.
of the Union Government.
For example, a state may offer
concession in fees for education to its (b) Government and legislature of states,
residents. that is, executive and legislative organs
of state Government.
(2) The Parliament (under Article 16) can
prescribe residence within a state or (c) All local authorities that is,
union territory as a condition for certain municipalities, panchayats, district
employments or appointments in that boards, improvement trusts, etc.
state or union territory, or local (d) All other authorities, that is, statutory
authority or other authority within that or non-statutory authorities like LIC,
state or union territory. Accordingly, the ONGC, SAIL, etc.
Parliament enacted the Public Thus, State has been defined in a wider
Employment (Requirement as to sense so as to include all its agencies. It is
[TEST- 4 EXPLANATION] [KSG: 011-45552607, 47640303, 0141-4052441, 0755-4077441, 09386337412] Page 8

Googlw it: pdf4exams


https://t.me/pdf4exams www.pdf4exams.in

KSG: MEMBER’S COPY


the actions of these agencies that can be Explanation:
challenged in the courts as violating the Statement 2 is incorrect
Fundamental Rights. In the Indian System, the constitution is the
According to the Supreme Court, even a source of the individual rights.
private body or an agency working as an Statement 1 and 3 are correct
instrument of the State falls within the
Rule of Law The concept of ‗equality before
meaning of the ‗State‘ under Article 12.
law‘ is an element of the concept of ‗Rule of
Source:LaxmikanthVth edition, Chapter 7, Law‘, propounded by A.V. Dicey, the British
page no 7.4 jurist. His concept has the following three
elements or aspects:
24. Consider the following statements: (i) Absence of arbitrary power, that is, no
1. Right to practice any Profession is man can be punished except for a
available to both citizens and breach of law.
foreigners. (ii) Equality before the law, that is, equal
2. Right to protection of script, subjection of all citizens (rich or poor,
language and culture is available high or low, official or non-official) to
to citizens of India only. the ordinary law of the land
3. Freedom to manage religious administered by the ordinary law
affairs is available to citizens of courts.
India only. (iii) The primacy of the rights of the
Which of the statements given above individual, that is, the constitution is
is/are correct? the result of the rights of the individual
(a) 1 only as defined and enforced by the courts
of law rather than the constitution
(b) 1 and 3 only
being the source of the individual
(c) 2 only rights.
(d) All of the above The Supreme Court held that the ‗Rule of
Explanation: Law‘ as embodied in Article 14 is a ‗basic
Statement 1 and 3 are incorrect feature‘ of the constitution. Hence, it cannot
Right to practice any Profession is available be destroyed even by an amendment.
to citizens of Indiaonly. Source:LaxmikanthVth edition, Chapter 7,
Freedom to manage religious affairs is page no 7.9
available to both citizens and foreigners.
Statement 2 is correct 26. Which of the following is not the
Protection of language, script and culture of exception to Article 15 of general rule
minorities (Article 29) is available only to the of non-discrimination?
citizens of India. (a) The state is permitted to make any
Source:LaxmikanthVth edition, Chapter 7, special provision for women and
page no 7.6 children.
(b) Fee concessions in public
25. Consider the following statements with educational institutions for socially
respect to Rule of Law: and educationally backward
classes.
1. It means no man can be punished
except for a breach of law. (c) The State can provide for
reservation of appointments or
2. In Indian system, constitution is
posts in favour of any backward
the result of the rights of the
class that is not adequately
individual.
represented in the state
3. It is the basic feature of the services.
constitution.
(d) None of the above
Which of the statements given above
Explanation:
is/are correct?
Statement ‗a‘ and ‗b‘ are an exception
(a) 1 only
(a) The state is permitted to make any
(b) 1 and 3 only
special provision for women and
(c) 2 and 3 only children. For example, reservation of
(d) 2 only seats for women in local bodies or
provision of free education for children.
[TEST- 4 EXPLANATION] [KSG: 011-45552607, 47640303, 0141-4052441, 0755-4077441, 09386337412] Page 9

Googlw it: pdf4exams


https://t.me/pdf4exams www.pdf4exams.in

KSG: MEMBER’S COPY


(b) The state is permitted to make any The ‗Face of Disasters 2019‘ report had been
special provision for the advancement of released by SEEDS (Sustainable
any socially and educationally backward Environment and Ecological Development
classes of citizens or for the scheduled Society).
castes and scheduled tribes. For SEEDS is a non-profit voluntary organisation
example, reservation of seats or fee that works towards building resilience in
concessions in public educational vulnerable communities. Its workforce
institutions. comprises young professionals drawn from
Statement c is not an exception various development related fields.
The State can provide for reservation of Statement 2 is correct
appointments or posts in favour of any Analysis of past trends shows us that 2019
backward class that is not adequately will see unusual flooding, as well as
represented in the state services. heatwaves and drought that are already
This is an exception to Article 16. ongoing.
Source:LaxmikanthVth edition, Chapter 7, Source:KSG Current Connect, April 2019,
page no 7.10 Page no 23, 24

27. ‗CheckPointTipline‘ was in news 29. What were provisions held by the
recently is: Supreme Court in the famous Mandal
(a) Electronic toll collection on Toll case 1992?
plazas on National Highways. 1. Apex Court rejected the additional
(b) E-filling of income tax by an reservation of 10% for poorer
individual. sections of higher castes.
(c) Platform launched by Whatsapp 2. Carry forward rule in case of
to address the issue of fake news backlog vacancies is not valid.
during recentLokSabha 3. A permanent statutory body
elections. should be established to examine
(d) None of the above complaints of over-inclusion and
Explanation: under-inclusion in the list of
OBCs.
Facebook-owned WhatsApp has launched a
new platform to address the issue of fake Select the correct answer using the
news duringLokSabha elections called code given below:
‗CheckPointTipline‘. (a) 1 and 2 only
The goal of this project is to study the (b) 1 and 3 only
misinformation phenomenon at scale — (c) 2 and 3 only
natively in WhatsApp — during the Indian (d) 1, 2 and 3
elections. Explanation:
Source:KSG Current Connect, April 2019, Statement 1 is correct
page no 21
In the famous Mandal case (1992), the scope
and extent of Article 16(4), which provides
28. Consider the following statements for reservation of jobs in favour of backward
about the ‗Face of Disasters‘ report: classes, has been examined thoroughly by
1. It is released by the United Nations the Supreme Court. Though the Court has
Environment Program. rejected the additional reservation of 10% for
2. Its recent report stated that 2019 poorer sections of higher castes.
will witness unusual flooding. Statement 2 is incorrect
Which of the statements given above The SC stated that the, ‗carry forward rule‘ in
is/are correct? case of unfilled (backlog) vacancies is valid.
(a) 1 only But it should not violate 50% rule.
(b) 2 only Statement 3 is correct
(c) Both 1 and 2 A permanent statutory body should be
(d) Neither 1 nor 2 established to examine complaints of over-
inclusion and under-inclusion in the list of
Explanation:
OBCs.
Statement 1 is incorrect
Source:LaxmikanthVth edition, Chapter 7,
page no 7.13

[TEST- 4 EXPLANATION] [KSG: 011-45552607, 47640303, 0141-4052441, 0755-4077441, 09386337412] Page 10

Googlw it: pdf4exams


https://t.me/pdf4exams www.pdf4exams.in

KSG: MEMBER’S COPY


30. According to Article 19 of the 6.Right to freedom from noise
Constitution of India, reasonable pollution.
restrictions can be imposed on the Select the correct answer using the
fundamental rights. On which of the code given below:
following rights a reasonable restriction (a) 2, 4 and 5 only
can be imposed in the interest of
(b) 1, 2, 3 and 5 only
Sovereignty and Integrity of India and
Public order? (c) 1, 3 and 6 only
1. Right to know about government (d) 1, 2, 3, 4, 5 and 6
activities. Explanation:
2. Right to reside in any part of the Some more rights which have been inferred
country. from Right to Life are:
3. Right to hold public meetings. (1) Right of appeal from a judgement of
4. Right to carry on any occupation. conviction
5. Right to freedom of commercial (2) Right to social security and protection
advertisements. of the family Right to social and
economic justice and empowerment.
Select the correct answer using the
code given below: (3) Right against bar fetters
(a) 1, 2 and 3 only (4) Right to appropriate life insurance
policy
(b) 4 and 5 only
(5) Right to sleep
(c) 1, 3 and 5 only
(6) Right to freedom from noise pollution
(d) 1, 2, 3, 4 and 5
Source:LaxmikanthVth edition, Chapter no
Explanation:
7, page no 7.22, 7.23
Statement 1 and 5 are correct
Right to know about government activities
32. Consider the following statements with
and Right to freedom of commercial
reference to Directive Principles of
advertisements can be restricted on the
State Policy (DPSP):
grounds of sovereignty and integrity of India,
security of the state, friendly relations with 1. They are the instructions to the
foreign states, public order, decency or legislature and the executive.
morality, contempt of court, defamation, and 2. They are justiciable in nature.
incitement to an offence. 3. They are fundamental in the
Statement 2 is incorrect governance of the country.
Right to reside in any part of the country can Which of the statements given above
be restricted in the interest of general public is/are correct?
and the protection of interests of any (a) 1only
scheduled tribes. (b) 1 and 3 only
Statement 3 is correct (c) 2 only
Right to hold public meetings can be (d) All of the above
restricted in the interest of sovereignty and Explanation:
integrity of India and public order.
Statement 1 is correct
Statement 4 is incorrect
The Directive Principles resemble the
Right to carry on any occupation can be ‗Instrument of Instructions‘ enumerated in
restricted in the interest of the general the Government of India Act of 1935. What is
public. called Directive Principles is merely another
Source:LaxmikanthVth edition, Chapter 7, name for the instrument of instructions. The
page no 7.17 only difference is that they are instructions
to the legislature and the executive‘.
31. Which of the following rights have been Statement 2 is incorrect
inferred from Article 21(Right to Life)? The Directive Principles are non-justiciable
1. Right against handcuffing in nature, that is, they arenot legally
2. Right to emergency medical aid enforceable by the courts for their violation.
3. Right against delayed execution Statement 3 is correct
4. Right to hearing The Constitution (Article 37) itself says that
5. Right against public hanging these principles are fundamental in the
governance of the country and it shall be the
[TEST- 4 EXPLANATION] [KSG: 011-45552607, 47640303, 0141-4052441, 0755-4077441, 09386337412] Page 11

Googlw it: pdf4exams


https://t.me/pdf4exams www.pdf4exams.in

KSG: MEMBER’S COPY


duty of the State to apply these principles in 34. Which of the statements given below
making laws. are fundamental duties of the Indian
Source:LaxmikanthVth edition, Chapter 8, citizen as enshrined in the Article 51-A
page no 8.2 of the Indian Constitution?
1. To promote harmony and the spirit
33. Consider the following statements of common brotherhood.
about Financial Emergency: 2. To develop scientific temper.
1. With the proclamation of 3. Promotion of international peace
Financial Emergency, salaries of and security
judges of Supreme Court can be 4. To safeguard public property.
reduced. Select the correct answer using the
2. A resolution approving the code given below:
proclamation of financial (a) 1,2 and 3 only
emergency can be passed by (b) 1, 2 and 4 only
either House of Parliament only
(c) 2, 3 and 4 only
by a special majority.
(d) 1,3 and 4 only
Which of the statements given above
is/are correct? Explanation:
(a) 1 only According to Article 51 A, it shall be the duty
of every citizen of India:
(b) 2 only
(a) to abide by the Constitution and
(c) Both 1 and 2
respect its ideals and institutions, the
(d) Neither 1 nor 2 National Flag and the National
Explanation: Anthem;
Statement 1 is correct: (b) to cherish and follow the noble ideals
The consequences of the proclamation of a that inspired the national struggle for
Financial Emergency are as follows: freedom;
1. The executive authority of the Centre (c) to uphold and protect the sovereignty,
extends (a) to directing any state to unity and integrity of India;
observe such canons of financial (d) to defend the country and render
propriety as are specified by it; and (b) national service when called upon to do
to directions as the President may so;
deem necessary and adequate for the (e) to promote harmony and the spirit of
purpose. common brotherhood amongst all the
2. Any such direction may include a people of India transcending religious,
provision requiring (a) the reduction of linguistic and regional or sectional
salaries and allowances of all or any diversities and to renounce practices
class of persons serving in the state; derogatory to the dignity of women;
and (b) the reservation of all money (f) to value and preserve the rich heritage
bills or other financial bills for the of the country‘s composite culture;
consideration of the President after
(g) to protect and improve the natural
they are passed by the legislature of
environment including forests, lakes,
the state.
rivers and wildlife and to have
3. The President may issue directions for compassion for living creatures;
the reduction of salaries and
(h) to develop scientific temper, humanism
allowances of (a) all or any class of
and the spirit of inquiry and reform;
persons serving the Union; and (b) the
judges of the Supreme Court and the (i) to safeguard public property and to
high court. abjure violence;
Statement 2 is incorrect: (j) to strive towards excellence in all
spheres of individual and collective
A resolution approving the proclamation of
activity so that the nation constantly
financial emergency can be passed by either
rises to higher levels of endeavour and
House of Parliament only by a simple
achievement; and
majority, that is, a majority of the members
of that house present and voting. (k) to provide opportunities for education
to his child or ward between the age of
Source:laxmikant Polity 5th edition,
six and fourteen years. This duty was
Chapter: 16, Financial Emergency

[TEST- 4 EXPLANATION] [KSG: 011-45552607, 47640303, 0141-4052441, 0755-4077441, 09386337412] Page 12

Googlw it: pdf4exams


https://t.me/pdf4exams www.pdf4exams.in

KSG: MEMBER’S COPY


added by the 86th Constitutional (b) 1, 2, 5 and 6 only
Amendment Act, 2002. (c) 1, 2, 3 and 4 only
Promotion of international peace and (d) 1,2, 3, 4, 5 and 6
security is one of the DPSPs. Explanation:
Source:LaxmikanthVth edition, Chapter 9, A number of provisions in the Constitution
page no 9.2 and 9.3 can be amended by a simple majority of the
two Houses of Parliament outside the scope
35. In accordance with the provisions of Article 368.
under Article 368 of the Constitution of These provisions include:
India, which of the following Second Schedule—emoluments, allowances,
statements is/are true? privileges and so on of the president, the
1. A bill seeking an amendment of governors, the Speakers, judges, etc.
the constitution can be introduced Fifth Schedule—administration of
in either house of the parliament. scheduled areas and scheduled tribes
2. The bill cannot be introduced by a Conferment of more jurisdiction on the
private member. Supreme Court
3. The bill doesn‘t require the prior Elections to Parliament and state
permission of the President. legislatures.
Select the correct answer using the Salaries and allowances of the members of
code given below: Parliament
(a) 1 only Quorum in Parliament
(b) 1 and 3 only Source:LaxmikanthVth edition, Chapter 10,
(c) 3 only page no 10.3
(d) 1, 2 and 3
Explanation: 37. Consider the following statements:
Statement 1 is correct 1. In the KesavanandaBharti case,
An amendment of the Constitution can be Supreme Court laid down a new
initiated only by the introduction of a bill for doctrine of the ‗Basic structure‘ of
the purpose in either House of Parliament the constitution.
and not in the state legislatures. 2. What constitutes a basic structure
Statement 2 is incorrect is clearly mentioned in the
The bill can be introduced either by a constitution of India.
minister or by a private member Which of the statements given above
Statement 3 is correct is/are correct?
It does not require prior permission of the (a) 1 only
president. (b) 2 only
Source:LaxmikanthVth edition, Chapter 10, (c) Both 1 and 2
page no 10.1, 10.2 (d) Neither 1 nor 2
Explanation:
36. Which of the following provisions in the Statement 1 is correct
constitution can be amended by a In the KesavanandaBharati case (1973), the
Simple majority of the two houses of Supreme Court overruled its judgement in
the parliament? the GolakNath case (1967). It upheld the
1. Second schedule validity of the 24th Amendment Act (1971)
2. Fifth Schedule and stated that Parliament is empowered to
3. Conferment of more jurisdiction on abridge or take away any of the Fundamental
the Supreme Court Rights. At the same time, it laid down a new
4. Elections to Parliament and State doctrine of the ‗basic structure‘ (or ‗basic
legislature features‘) of the Constitution. It ruled that
the constituent power of Parliament under
5. Salaries and allowances of the
Article 368 does not enable it to alter the
member of the parliament
‗basic structure‘ of the Constitution. This
6. Quorum in Parliament means that the Parliament cannot abridge or
Select the correct answer using the take away a Fundamental Right that forms a
code given below: part of the ‗basic structure‘ of the
(a) 3,4,5 and 6 only Constitution.
[TEST- 4 EXPLANATION] [KSG: 011-45552607, 47640303, 0141-4052441, 0755-4077441, 09386337412] Page 13

Googlw it: pdf4exams


https://t.me/pdf4exams www.pdf4exams.in

KSG: MEMBER’S COPY


Statement 2 is incorrect (a) 1,2 and 4 only
What constitutes a Basic Structure is not (b) 2, 3 and 4 only
clearly mentioned in the constitution of (c) 1 and 3 only
India. (d) 1, 2, 3 and 4
Parliament under Article 368 can amend any Explanation:
part of the Constitution including the
Statement 1 is correct
Fundamental Rights but without affecting
the ‗basic structure‘ of the Constitution. Leadership of the Prime Minister: The Prime
However, the Supreme Court is yet to define Minister plays the leadership role in this
or clarify as to what constitutes the ‗basic system of government. He is the leader of
structure‘ of the Constitution. council of ministers, leader of the Parliament
and leader of the party in power. In these
Source:LaxmikanthVth edition, Chapter 11,
capacities, he plays a significant and highly
page no 11.2
crucial role in the functioning of the
government.
38. Consider the following statements Statement 2 is correct
about the Parliamentary form of
Majority Party Rule: The political party which
Government in India:
secures majority seats in the LokSabha
1. The executive is responsible to the forms the government. The leader of that
Legislature. party is appointed as the Prime Minister by
2. Only members elected in the President; other ministers are appointed
Parliament are eligible to become a by the President on the advice of the prime
Minister. minister.
Which of the statements given above Statement 3 is incorrect
is/are correct? There is no clear separation of power
(a) 1 only between legislature and executive. The
(b) 2 only greatest advantage of the parliamentary
(c) Both 1 and 2 system is that it ensures harmonious
(d) Neither 1 nor 2 relationship and cooperation between the
legislative and executive organs of the
Explanation:
government. The executive is a part of the
Statement 1 is correct legislature and both are interdependent at
The parliamentary system of government is work. As a result, there is less scope for
the one in which the executive is responsible disputes and conflicts between the two
to the legislature for its policies and acts. organs.
Statement 2 is incorrect The doctrine of separation of powers is the
The ministers are members of both the basis of the American presidential system.
legislature and the executive. This means The legislative, executive and judicial powers
that a person cannot be a minister without of the government are separated and vested
being a member of the Parliament. The in the three independent organs of the
Constitution stipulates that a minister who government.
is not a member of the Parliament for a Statement 4 is correct
period of six consecutive months ceases to be Secrecy: The ministers operate on the
a minister. principle of secrecy of procedure and cannot
Source:LaxmikanthVth edition, Chapter 12, divulge information about their proceedings,
page no: 12.1 &12.3 policies and decisions. They take the oath of
secrecy before entering their office. The oath
39. Which of the following are the of secrecy to the ministers is administered by
principles on which Parliamentary form the President.
of Government operates? Source:LaxmikanthVth edition, chapter 12,
1. Leadership of the Prime Minister page 12.1, 12.2, 12.3, 12.4 and 12.5
2. Majority Party Rule
3. Clear separation of powers 40. Consider the following provisions or
between Legislature and executive. features:
4. Principle of Secrecy 1. For the first time the political and
Select the correct answer using the administrative functions of the
code given below: company were recognised.

[TEST- 4 EXPLANATION] [KSG: 011-45552607, 47640303, 0141-4052441, 0755-4077441, 09386337412] Page 14

Googlw it: pdf4exams


https://t.me/pdf4exams www.pdf4exams.in

KSG: MEMBER’S COPY


2. It made the governors of Bombay 41. Consider the following pairs:
and Madras presidencies List I List II
subordinate to the governor-
general of Bengal. 1. Lord Warren First governor
Hasting general of India
3. It provided for the establishment of
a Supreme Court at Calcutta. 2. Lord William First governor
4. It prohibited servants of the Bentick general of
company from engaging in any Bengal.
private trade. 3. Lord Canning First Viceroy of
Identify the act with the help of above India
statements: Which of the pairs given above is/are
(a) Regulating Act 1773 correctly matched?
(b) Pitts India Act 1784 (a) 1 only
(c) Charter Act 1813 (b) 2 and 3 only
(d) Charter Act 1833 (c) 3 only
Explanation: (d) 1, 2 and 3
Regulating Act of 1773 Explanation:
This act is of great constitutional importance Lord Warren Hasting was the first governor
as general of Bengal.
(a) It was the first step taken by the Lord William Bentick was the first governor
British Government to control and general of India.
regulate the affairs of the East India Lord Canning was the first Viceroy of India
Company in India; Source:LaxmikanthVth edition, Chapter 1,
(b) It recognised, for the first time, the page no 1.2, 1.3 &1.5
political and administrative functions
of the Company; and
42. Consider the following statements
(c) It laid the foundations of central regarding the Constituent Assembly:
administration in India.
1. It was based on the Cabinet
Features of the Act Mission Plan.
1. It designated the Governor of Bengal as 2. Representatives of princely state
the ‗Governor-General of Bengal‘ and were nominated by the Governor
created an Executive Council of four General.
members to assist him. The first such
3. It was wholly a nominated body.
Governor-General was Lord Warren
Hastings. Which of the statements given above
is/are correct?
2. It made the governors of Bombay and
Madras presidencies subordinate to the (a) 1 only
governor- general of Bengal, unlike (b) 1 and 2 only
earlier, when the three presidencies were (c) 3 only
independent of one another. (d) 1, 2 and 3
3. It provided for the establishment of a Explanation:
Supreme Court at Calcutta (1774) Statement 1 is correct
comprising one chief justice and three
The Constituent Assembly was constituted in
other judges.
November 1946 under the scheme
4. It prohibited the servants of the formulated by the Cabinet Mission Plan.
Company from engaging in any private
Statement 2 is incorrect
trade or accepting presents or bribes
from the ‗natives‘. The representatives of princely states were to
be nominated by the heads of the princely
5. It strengthened the control of the British
states.
Government over the Company by
requiring the Court of Directors The total strength of the Constituent
(governing body of the Company) to Assembly was to be 389. Of these, 296 seats
report on its revenue, civil, and military were to be allotted to British India and 93
affairs in India. seats to the Princely States. Out of 296 seats
allotted to the British India, 292 members
Source:LaxmikanthVth edition, Chapter 1,
were to be drawn from the eleven governors‘
page no 1.2

[TEST- 4 EXPLANATION] [KSG: 011-45552607, 47640303, 0141-4052441, 0755-4077441, 09386337412] Page 15

Googlw it: pdf4exams


https://t.me/pdf4exams www.pdf4exams.in

KSG: MEMBER’S COPY


provinces and four from the four Chief 2. It will work as a catalyst for India‘s
Commissioners‘ provinces, one from each. ‗Act East‘ policy.
Statement 3 is incorrect 3. Recently China has included this
The representatives of each community were initiative in the list of projects
to be elected by members of that community covered under the Belt and Road
in the provincial legislative assembly and Initiative (BRI).
voting was to be by the method of Which of the statements given above
proportional representation by means of is/are correct?
single transferable vote. (a) 1 only
It is thus clear that the Constituent (b) 1 and 2 only
Assembly was to be a partly elected and (c) 3 only
partly nominatedbody. Moreover, the
(d) 1, 2 and 3
members were to be indirectly elected by the
members of the provincial assemblies, who Explanation:
themselves were elected on a limited Statement 1 is correct
franchise. It is the proposed 2,800-km long Economic
Source:LaxmikanthVth edition, Chapter 2, Corridor that will connect Kolkata with
page no 2.2 Kunming (Yunnan Province).  It is the
result of decades old discussion under
Track-II process known as the 'Kunming
43. Consider the following statements with
Initiative'.
respect to Objective Resolution:
Statement 2 is correct
1. It was moved by Jawaharlal Nehru
in the assembly. The corridor goes beyond physical
connectivity through to providing digital
2. All power and authority of
connectivity, trade facilitation and lowering
Sovereign Independent India were
barriers for smooth and seamless movement
to be derived from the people.
of goods, services, investment and people. It
3. It was unanimously adopted by the has the potential to generate huge economic
assembly just after India‘s benefits in trade, investment, energy,
independence. transport and communication linkages. It
Which of the statements given above will open-up markets for India in the east,
is/are correct? which will also be pivotal for India‘s ‗Act
(a) 1only (b) 1 and 2 only East‘ policy.
(c) 3 only (d) 1, 2 and 3 It will help India in opening-up of the
Explanation: economic potential of the North-Eastern
Statement 1 is correct: states.
On December 13, 1946, Jawaharlal Nehru Statement 3 is incorrect
moved the historic ‗Objectives Recently China has excluded Bangladesh-
Resolution‘ in the Assembly. It laid down the China-India-Myanmar (BCIM) Economic
fundamentals and philosophy of the corridor from the list of projects covered by
constitutional structure. the China-led Belt and Road Initiative (BRI)
umbrella.
Statement 2 is correct: All power and
authority of the Sovereign Independent India, Source:KSG Current Connect, April 2019,
its constituent parts and organs of page no 28
Government are derived from the people.
Statement 3 is incorrect: This Resolution 45. Consider the following statements with
was unanimously adopted by the Assembly regard to National Emergency
on January 22, 1947. according to the constitution of India:
Source:LaxmikanthVth edition, Chapter 2, 1. The legislative power of a state
page 2.3 &2.4 legislature is suspended.
2. Modification done by the President
44. Consider the following statements in the constitutional distribution of
about the Bangladesh-China-India- revenues between the center and
Myanmar (BCIM) Economic corridor: the state are void immediately after
the Emergency ceases to operate.
1. It is the result of decades old
discussion under Track-II process
known as the 'Kunming Initiative.

[TEST- 4 EXPLANATION] [KSG: 011-45552607, 47640303, 0141-4052441, 0755-4077441, 09386337412] Page 16

Googlw it: pdf4exams


https://t.me/pdf4exams www.pdf4exams.in

KSG: MEMBER’S COPY


Which of the statements given above (d) He must give his assent
is/are correct? Explanation:
(a) 1 only If a Constitutional Amendment bill is
(b) 2 only presented to the President for assent. The
(c) Both 1 and 2 president must give his assent to the bill. He
(d) Neither 1 nor 2 can neither withhold his assent to the bill
nor return the bill for reconsideration of the
Parliament.
Explanation:
Source:LaxmikanthVth edition, Chapter 10,
Both the statements are incorrect: page no 10.2

Statement 1: During a national emergency, 48. Consider the following statements:


the Parliament becomesempowered to make 1. According to the constitution of
laws on any subject mentioned in the State India, under President Rule, all
List.Although the legislative power of a Fundamental Rights are
state legislature is not suspended, suspended except the rights which
itbecomes subject to the overriding power of are guaranteed under Article 20
the Parliament. and 21.
2. During Financial emergency, The
Statement 2: While a proclamation of President may issue directions for
national emergency is in operation, the the reduction of salaries and
President can modify the constitutional allowances of the judges of the
distribution of revenues between the centre Supreme Court.
and the states. This means that the
3. President Rule can be revoked by
president can either reduce or cancel the
the President only on his own.
transfer of finances from Centre to the
states. Such modification continues till Which of the statements given above
the end of the financial year in which the is/are correct?
Emergency ceases to operate. Also, every (a) 1 only
such order of the President has to be laid (b) 1 and 2 only
before both the Houses of Parliament. (c) 2 and 3 only
(d) 3 only
Source: Laxmikanth, Vth edition, Chapter
16(emergency provisions), page 16.5&16.6
Explanation:

46. Which of the actsgiven below separated Statement 1 is incorrect:President Rule


for the first time the provincial budget has no effect on the fundamental rights
from the central budget? enshrined in the Indian Constitution. During
(a) Indian Council Act 1892 National Emergency, Article 359 deals with
(b) Indian Council act 1909 the suspension of other Fundamental Rights
(c) Government of India Act 1919 except those guaranteed by Articles 20 and
21.
(d) Government of India Act 1935
Explanation: Statement 3 is correct: President Rule
Government of India act 1919: It separated, can be revoked by the President only on
for the first time, provincial budgets from the his own. There is no such provision of
Central budget and authorised the provincial requirement of LokSabha to pass a
legislatures to enact their budgets. resolution for its revocation.
Source:LaxmikanthVth edition, Chapter 1,
page no 8 Statement 2 is correct: The President may
issue directions for the reduction of salaries
47. If a Constitutional Amendment bill is and allowances of (a) all or any class of
presented to the President for assent. persons serving the Union; and (b) the
The President: judges of the Supreme Court and the high
court.
(a) May give his assent
(b) May withhold his assent
(c) May return the bill for
reconsideration of the parliament
[TEST- 4 EXPLANATION] [KSG: 011-45552607, 47640303, 0141-4052441, 0755-4077441, 09386337412] Page 17

Googlw it: pdf4exams


https://t.me/pdf4exams www.pdf4exams.in

KSG: MEMBER’S COPY


Source: Laxmikanth, Vth edition, Chapter ministers and assist them in the discharge of
16(emergency provisions), page 16.13, their parliamentary duties.
16.14&16.21 Source:LaxmikanthVth edition, Chapter 20,
page no 20.8
49. Consider the following statements:
1. The Cabinet decisions are binding 51. Consider the following statements
on all the central ministers. about the Council of Ministers:
2. There is no provision in the 1. Their advices to the President are
Constitution for the system of legal binding in nature.
responsibility of a minister. 2. They cease to hold the office with
Which of the statements given above the dissolution of the LokSabha.
is/are correct? Which of the statements given above
(a) 1 only is/are correct?
(b) 2 only (a) 1 only
(c) Both 1 and 2 (b) 2 only
(d) Neither 1 nor 2 (c) Both 1 and 2
Explanation: (d) Neither 1 nor 2
Statement 1 is correct Explanation:
The principle of collective responsibility also Statement 1 is correct
means that the Cabinet decisions bind all Article 74 provides for a council of ministers
cabinet ministers (and other ministers) even with the Prime Minister at the head to aid
if they differed in the cabinet meeting. and advise the President in the exercise of
Statement 2 is correct his functions. The 42nd and 44th
In India there is no provision in the Constitutional Amendment Acts have made
Constitution for the system of legal the advice binding on the President.
responsibility of a minister. It is not required Statement 2 is incorrect
that an order of the President for a public act In 1971, the Supreme Court held that ‗even
should be countersigned by a minister. after the dissolution of the LokSabha, the
Moreover, the courts are barred from council of ministers does not cease to hold
enquiring into the nature of advice rendered office.
by the ministers to the president. Source:LaxmikanthVth edition, Chapter 20,
Source:LaxmikanthVth edition, Chapter 6 & page no 20.3
7
52. Consider the following Statements
50. Consider the following statements about Article 123-
about the Parliamentary Secretaries:
1. They are the members of the last 1.
Article 123 have similar effect as
category of the council of an act of the Parliament.
ministers. 2. It is not accounted as a Legislative
2. They could be given the charge of process.
separate ministries. 3. It is outside the purview of Judicial
Which of the statements given above Review.
is/are correct? Which of the statements given above
(a) 1 only is/are correct?
(b) 2 only (a) 1 Only
(c) Both 1 and 2 (b) 1 and 2 Only
(d) Neither 1 nor 2 (c) 2 and 3 Only
Explanation: (d) 1, 2 and 3
Statement 1 is correct Explanation:
They are the members of the last category of
Statement 1 is correct.
the council of ministers (which is also known
as the ‗ministry‘). Article 123 of the Constitution
Statement 2 is incorrect empowers the President to promulgate
They have no department under their ordinances during the recess of
control. They are attached to the senior Parliament. These ordinances have
[TEST- 4 EXPLANATION] [KSG: 011-45552607, 47640303, 0141-4052441, 0755-4077441, 09386337412] Page 18

Googlw it: pdf4exams


https://t.me/pdf4exams www.pdf4exams.in

KSG: MEMBER’S COPY


the same force and effect as an act of Article 72 of the Constitution empowers the
Parliament, but are in the nature of President to grant pardons to persons who
temporary laws. have been tried and convicted of any offence
in all cases where the: 1. Punishment or
Statement 2 is correct. sentence is for an offence against a Union
Law; 2. Punishment or sentence is by a court
The ordinance-making power is the most martial (military court); and 3. Sentence is a
important legislative power of sentence of death.
the President. Legislative process is
Statement 3 is incorrect
purely belongs to parliamentary process
which is followed in enactment of act. The Supreme Court examined the pardoning
Though parliamentary sanction required for power of the President under different cases
ordinance to convert it into act, but separate and laid down the following principles: 1.
ordinance is an ‘executive exercise’ rather The petitioner for mercy has no right to an
than parliamentary process. oral hearing by the President. 2. The
So ordinance don’t coincides as pure President can examine the evidence afresh
legislative process as followed in passing of and take a view different from the view taken
certain bills. by the court. 3. The power is to be exercised
by the President on the advice of the union
Statement 3 is incorrect. cabinet. 4. The President is not bound to give
reasons for his order.
An ordinance is subject to the same Source:LaxmikanthVth edition, Chapter 17,
constitutional limitation as an act of page no 17.20 &17.21
Parliament. Hence, an ordinance cannot
abridge or take away any of
54. Consider the following statements
the fundamental rights or it must not
about the Vice-President of India:
affect Basic structure of constitution.
Hence it is in the purview of Judicial 1. He is elected by all the Members of
Review. Parliament.
2. He acts as the ex-officio chairman
Source- M Laxmikant Chapter- 17 of the RajyaSabha.
Ordinance making power of president 3. The Constitution has not fixed any
emoluments for the Vice-President
in that capacity.
53. Consider the following statements Which of the statements given above
about the pardoning power of the is/are correct?
President: (a) 1 only
1. While exercising this power the (b) 1 and 2 only
President sits as a court of appeal.
(c) 2 and 3 only
2. He can pardon sentences inflicted
by court martial. (d) 1, 2 and 3
3. The petitioner for mercy has a Explanation:
right to an oral hearing by the Statement 1 is correct
President. The Vice-President occupies the second
Which of the statements given above highest office in the country. He is accorded
is/are correct? a rank next to the President in the official
(a) 1 only warrant of precedence.
(b) 1 and 3 only The Electoral College for the election of the
Vice-President consists of both elected and
(c) 2 only nominated members of the Parliament (in the
(d) 1, 2 and 3 case of president, only elected members).
Explanation: Statement 2 is correct
Statement 1 is incorrect He acts as the ex-officio Chairman of
The pardoning power of the President is RajyaSabha. In this capacity, his powers and
independent of the Judiciary; it is an functions are similar to those of the Speaker
executive power. The President while of LokSabha. In this respect, he resembles
exercising this power, does not sit as a court the American vice-president who also acts as
of appeal. the Chairman of the Senate—the Upper
Statement 2 is correct House of the American legislature.

[TEST- 4 EXPLANATION] [KSG: 011-45552607, 47640303, 0141-4052441, 0755-4077441, 09386337412] Page 19

Googlw it: pdf4exams


https://t.me/pdf4exams www.pdf4exams.in

KSG: MEMBER’S COPY


Statement 3 is correct Explanation:
The Constitution has not fixed any Supremacy of the Constitution is one of the
emoluments for the Vice-President in that Federalfeaturesof the Constitution.
capacity. He draws his regular salary in his The Unitary principles which are mentioned
capacity as the ex-officio Chairman of the above are found in Indian Federation are:
RajyaSabha. Strong Centre
Source:LaxmikanthVth edition, Chapter 18, States Not Indestructible
page no 18.1, 18.5 &18.6
Single Constitution
Flexibility of the Constitution
55. Consider the following statements:
No Equality of State Representation
1. Zonal Councils are constitutional Emergency Provisions
bodies. Single Citizenship
2. Prime Minister acts as a common Integrated Judiciary
chairman of Zonal Councils. All-India Services
Which of the statements given above Integrated Audit Machinery
is/are correct? Parliament‘s Authority Over State List
(a) 1 only Appointment of Governor
(b) 2 only Integrated Election Machinery
(c) Both 1 and 2 Veto Over State Bills
(d) Neither 1 nor 2 Source:LaxmikanthVth edition, chapter 13,
Explanation: page 13.9 &13.10

Statement 1 is incorrect: 57. The World Economic Outlook Report is


published by:
Zonal Councils are Statutory bodies, not
(a) International Monetary Fund
constitutional bodies. They are established
(b) World Bank
by an Act of the Parliament that is, States
(c) World Economic Forum
Reorganisation Act of 1956.
(d) UNCTAD
Statement 2 is incorrect: Explanation:
International Monetary Fund (IMF) recently
The Home Minister acts as a common published World Economic Outlook - 2019. It
chairman of Zonal Councils. is a survey by the IMF usually published
twice a year.
Source: https://mha.gov.in/zonal-council Source: KSG Current Connect, April 2019,
page no 71

56. Which one of the following is/are the 58. The terms Currency Chest refers to:
Unitary principles found in Indian (a) Some sort of currency store
constitution? houses which acts as networks
1. Supremacy of the Constitution of currency distributions.
2. Appointment of Governor by the (b) Those Chest where currencies are
President printed.
3. Election commission conducts (c) The chest where reverse tranche is
election for both the Centre and stored in RBI.
the States. (d) The channel which was created
4. All India services exclusively to store the
5. Integrated Judiciary demonetised currencies.
Select the correct answer using the Explanation:
code given below: Currency chests are some sort of currency
(a) 2,3, 4 and 5 only store houses and acts as networks of
(b) 1, 3 and 5 only currency distributions. Main function of
currency chests is to store currency notes
(c) 1, 2 and 4 only
and coins.
(d) 1, 2, 3, 4 and 5

[TEST- 4 EXPLANATION] [KSG: 011-45552607, 47640303, 0141-4052441, 0755-4077441, 09386337412] Page 20

Googlw it: pdf4exams


https://t.me/pdf4exams www.pdf4exams.in

KSG: MEMBER’S COPY


Currency chests facilitate note supply. They Explanation:
help the RBI to undertake smooth supply of Statement 1 is incorrect
currency notes across the country. The cash The Fundamental Duties in the Indian
reserve ratios of the commercial banks are Constitution are inspired by the Constitution
also kept at currency chests. of erstwhile USSR. Notably, none of the
Source: KSG Current Connect, April 2019, Constitutions of major democratic countries
page no 69, 70 like USA, Canada, France, Germany,
Australia and so on specifically contain a list
59. Consider the following statements: of duties of citizens. Japanese Constitution
1. Fundamental duties can be is, perhaps, the only democratic Constitution
enforced through writ jurisdiction. in world which contains a list of duties of
citizens.
2. Fundamental duties are intended
to serve as a reminder to every Statement 2 is correct
citizen to observe basic norms of Unlike some of the Fundamental Rights
democratic conduct. which extend to all persons whether citizens
3. Fundamental duties are intended or foreigners, the Fundamental Duties are
to serve as a reminder to the confined to citizens only and do not extend to
Judiciary to administer justice foreigners.
properly. Source:LaxmikanthVth edition, Chapter 9,
Which of the statements given above page no 9.2 &9.3
is/are incorrect?
(a) 1 only 61. Which of the statement given below is
(b) 1 and 3 only correct?
(c) 2 and 3 only (a) Primacy is given to all the Directive
Principles contained in Part IV of
(d) 2 only
the Constitution over fundamental
Explanation: rights.
Statement 1 and 3 are incorrect (b) Primacy is given to all the
Fundamental duties are non-justiciable in fundamental rights conferred by
nature, so they can‘t be enforced through Article 14-32 of the constitution
writ jurisdiction. over directive principles.
They are also not intended to serve as a (c) Primacy is given to all the
reminder to the Judiciary to administer fundamental rights conferred in
justiceproperly. Part III of the constitution over
Statement 2 is correct directive principles.
They serve as a reminder to the citizens that (d) Primacy is given only to
while enjoying their rights, they should also directive principles in clause (b)
be conscious of duties they owe to their and (c) of Article 39 over
country, their society and to their fellow fundamental rights conferred by
citizens. Articles 14 and 19 of the
Source:LaxmikanthVth edition, Chapter 9, constitution.
page no 9.4 Explanation:
The 25th Amendment Act inserted a new
60. Consider the following statements with Article 31C which contained the following
respect to the fundamental duties two provisions:
enshrined in the constitution of India: 1. No law which seeks to implement the
1. They are inspired by the socialistic Directive Principles specified
constitution of Japan. in Article 39 (b) and (c) shall be void on
2. They are confined to the Indian the ground of contravention of the
citizens only. Fundamental Rights conferred by Article
14 (equality before law and equal
Which of the statements given above
protection of laws), Article 19 (protection
is/are correct?
of six rights in respect of speech,
(a) 1 only assembly, movement, etc or Article 31
(b) 2 only (right to property).
(c) Both 1 and 2 2. No law containing a declaration for
(d) Neither 1 nor 2 giving effect to such policy shall be

[TEST- 4 EXPLANATION] [KSG: 011-45552607, 47640303, 0141-4052441, 0755-4077441, 09386337412] Page 21

Googlw it: pdf4exams


https://t.me/pdf4exams www.pdf4exams.in

KSG: MEMBER’S COPY


questioned in any court on the ground The Constitution does not contain any
that it does not give effect to such a classification of Directive Principles.
policy. However, on the basis of their content and
Source:LaxmikanthVth edition, Chapter 8, direction, they can be classified into three
page no 8.10 broad categories, viz, socialistic, Gandhian
and liberal–intellectual.
62. Consider the following statements: The Gandhian principles are:
1. Parliament have the exclusive 1. To organise village panchayats and
power to make laws on residuary endow them with necessary powers and
subjects. authority to enable them to function as
2. Education comes under union units of self-government (Article 40).
list. 2. To promote cottage industries on an
Which of the statements given above individual or co-operation basis in rural
is/are correct? areas (Article 43).
(a) 1 only 3. To promote voluntary formation,
(b) 2 only autonomous functioning, democratic
(c) Both 1 and 2 control and professional management of
(d) Neither 1 nor 2 co-operative societies (Article 43(B).
Explanation: 4. To promote the educational and
economic interests of SCs, STs, and
Statement 1 is correct other weaker sections of the society and
to protect them from social injustice and
The power to make laws on residuary exploitation (Article 46).
subjects rests with Parliament 5. To prohibit the consumption of
intoxicating drinks and drugs which are
Source: Indian Polity by M Laxmikanth, injurious to health (Article 47).
Chapter 14, Centre – State Relations 6. To prohibit the slaughter of cows, calves
and other milch and draught cattle and
Statement 2 is incorrect to improve their breeds (Article 48).
Socialistic principles: To secure prevention
The 42nd Amendment Act of 1976 transferred
of concentration of wealth and means of
five subjects to Concurrent List from state production.
list They are Education, Forests, Weights Liberal- Intellectual principles: To protect
and Measures, Protection of wild animals monuments, places and objects of artistic or
and birds. historic interest.
Source:LaxmikanthVth edition, Chapter 8,
Source: Indian Polity by M Laxmikanth, page no 8.3 &8.4
Chapter 14, Centre-State Relations
64. Consider the following statements
63. Consider the following Directive
about the Asian Development Outlook:
Principles of State Policies:
1. It has been published by the World
1. To secure prevention of
Bank.
concentration of wealth and means
of production. 2. In its recent report, it has lowered
India‘s growth by 40 basis points
2. To organise village panchayats.
to 7.2% in fiscal year 2019-20.
3. To promote cottage industries.
Which of the statements given above
4. To protect monuments, places and is/are correct?
objects of artistic or historic
(a) 1 only
interest.
(b) 2 only
Which of the statements given above
are Gandhian principles? (c) Both 1 and 2
(a) 1 and 2 only (d) Neither 1 nor 2
(b) 2 and 3 only Explanation:
(c) 1 and 4 only Statement 1 is incorrect
(d) All of the above Asian Development Bank (ADB) recently
published its flagship publication, Asian
Explanation:
Development Outlook 2019.

[TEST- 4 EXPLANATION] [KSG: 011-45552607, 47640303, 0141-4052441, 0755-4077441, 09386337412] Page 22

Googlw it: pdf4exams


https://t.me/pdf4exams www.pdf4exams.in

KSG: MEMBER’S COPY


Statement 2 is correct Explanation:
It lowered India‘s growth by 40 basis points Statement 1 is correct
to 7.2% in fiscal year 2019-20 because of a The term ‗federation‘ has no where been used
slower-than-expected pickup in investment in the Constitution. Instead, Article 1 of the
demand. Constitution describes India as a ‗Union of
Source: KSG Current Connect, April 2019, States‘. According to Dr B R Ambedkar, the
page no 70 phrase ‗Union of States‘ has been preferred
to ‗Federation of States‘ to indicate two
65. Consider the following statements things: (i) the Indian federation is not the
result of an agreement among the states like
regarding Veto Power of the President
the American federation; and (ii) the states
of India: have no right to secede from the federation.
The federation is union because it is
1. The Power of Veto refers to the indestructible.
power of the legislature to
Statement 2 is correct
override any act of the executive.
2. This power is exercised by the The Constitution stipulates three types of
president to ensure that no law emergencies—national, state and financial.
has been made in a haste. During an emergency, the Central
3. The President enjoys a Qualified government becomes all powerful and the
Veto in case of a Money Bill. states go into the total control of the Centre.
Which of the statements given above It converts the federal structure into a
is/are incorrect? unitary one without a formal amendment of
(a)1 and 2 only the Constitution. This kind of transformation
(b) 3 only is not found in any other federation.
(c) 1 and 3 only Source:Laxmikanth, chapter 13, page 13.1
(d) 1, 2 and 3 and 13.3
Explanation:
67. Consider the following statements
Article 111 in India‘s Constitution governs about the Powers of the President of
the Veto powers of the President. The power India:
of Veto refers to the power of the executive 1. He is empowered to apply any act
to override any act of the legislature. This of the Parliament with some
power is exercised by the president as a modifications to any scheduled
Constitutional discretion to ensure that no area in the state.
law has been made in a haste. The Veto 2. He can direct that an act of
Parliament does not applyto a
power does not apply in the case of a
tribal areas of the state of
Constitutional Amendment Bill. The Indian Meghalaya, Tripura and Mizoram.
President does not enjoy Qualified veto Which of the statements given above
(Which can be overridden by a higher is/are correct?
majority) in case of any bill. (a) 1 only
(b) 2 only
Source: Laxmikant Chapter 17: President.
(c) Both 1 and 2
66. Consider the following statements:
(d) Neither 1 nor 2
1. The term ‗federation‘ has nowhere
been used in the Constitution. Explanation:
Statement 1 is incorrect
2. The emergency provisions in the
Indian Constitution can convert The governor is empowered to direct that an
the Federal State into a Unitary act of Parliament does not apply to a
State. scheduled area in the state or apply with
specified modifications and exceptions.
Which of the statements given above
is/are correct? Statement 2 is correct
(a) 1 only The President enjoys the same power with
(b) 2 only respect to tribal areas (autonomous districts)
in Meghalaya, Tripura and Mizoram.
(c) Both 1 and 2
Source:LaxmikanthVth edition, chapter 14,
(d) Neither 1 nor 2 page14.2

[TEST- 4 EXPLANATION] [KSG: 011-45552607, 47640303, 0141-4052441, 0755-4077441, 09386337412] Page 23

Googlw it: pdf4exams


https://t.me/pdf4exams www.pdf4exams.in

KSG: MEMBER’S COPY


The Global Report on Food Crisis is released
68. Consider the following statements annually by the Food Security Information
about AUSINDEX: Network (FSIN), led by the UN Food and
1. It is a bilateral maritime exercise Agriculture Organization (FAO), the World
between Indian and Australian Food Programme (WFP) and IFPRI.
Navy. Statement 2 is incorrect
2. It included a series of advanced The report found that the number of people
warfare drills in all three facing food crises has come down slightly
dimensions comprising anti- from 2017. However the number of countries
submarine warfare exercises, air affected has risen. Moreover, an additional
defense exercises and anti-surface 143 million people in another 42 countries
warfare exercises. are just one step away from facing acute
Which of the statements given above hunger.
is/are correct? Source: KSG Current Connect, April 2019,
(a) 1 only (b) 2 only page no 79
(c) Both 1 and 2 (d) Neither 1 nor 2
Explanation: 70. Consider the following statements
about Assam Riffles:
Both the statements are correct
1. It comes under the administrative
The Australian and Indian Navy had two-
control of the Ministry of Defence.
week long bilateral maritime exercise code-
named AUSINDEX. This was the third 2. It was formed during the 1948
edition of the exercise and included a series Indo-Pakistan war.
of advanced warfare drills in all three 3. It also operated in foreign land as
dimensions comprising antisubmarine part of the Indian Peace Keeping
warfare exercises, air defense exercises, anti- Force (IPKF) in past.
surface warfare exercises including live-fire Which of the statements given above
drills, replenishment at sea, and cross deck is/are correct?
flying. The bilateral exercise was aimed ―to (a) 1 only
strengthen and enhance mutual cooperation (b) 2 and 3 only
and interoperability between the IN and RAN,
(c) 3 only
providing opportunities for interaction and
exchange of professional views between the (d) All of the above
personnel of the two navies‖. The current Explanation:
edition of the exercise had the participation Statement 1 is incorrect
of the highest number. Assam Riffles, the 184-year-old paramilitary
Source: KSG Current Connect, April 2019, force of the country, is under the
page no 88 administrative control of the Ministry of
Home Affairs while the operational control
69. Consider the following statements lies with the Ministry of Defence.
aboutthe Global Report on Food Crisis: Statement 2 is incorrect
1. It is released annually by the Food The Assam Rifles came into being in 1835, as
Security Information Network a militia called the ‗Cachar Levy‘.
(FSIN), led by the UN Food and Statement 3 is correct
Agriculture Organization (FAO). The Post-Independence role of the Assam
2. As per this report, there is a Rifles continued to evolve ranging from
significant increase in the number conventional combat role during Sino-India
of people facing food crises from War 1962, operating in foreign land as part
2017. of the Indian Peace Keeping Force (IPKF) to
Which of the statements given above Sri Lanka in 1987 (Op Pawan) to
is/are correct? peacekeeping role in the North-Eastern areas
(a) 1 only of India in the face of growing tribal unrest
and insurgency wherein the maintenance of
(b) 2 only
law and order, countering insurgency and
(c) Both 1 and 2 reassuring the people of the region became
(d) Neither 1 nor 2 important tasks for the Assam Rifles.
Explanation: Source: KSG Current Connect, April 2019,
Statement 1 is correct page no 87

[TEST- 4 EXPLANATION] [KSG: 011-45552607, 47640303, 0141-4052441, 0755-4077441, 09386337412] Page 24

Googlw it: pdf4exams


https://t.me/pdf4exams www.pdf4exams.in

KSG: MEMBER’S COPY


governor of the state, can be removed only by
71. Consider the followings statements: the President.
1. In respect of matters in the Statement 2 is correct
Concurrent List, the executive The Parliament can establish a Joint State
power rests with the states, Public Service Commission (JSPSC) for two
although with exceptions. or more states on the request of the state
2. No tax shall be levied or collected legislatures concerned. The chairman and
except by authority of law. members of the JSPSC are appointed by the
Which of the statements given above president.
is/are incorrect? Source:LaxmikanthVth edition, chapter 14,
(a) 1 only page 14.11
(b) 2 only
(c) Both 1 and 2 73. Consider the following statements
about the Migration and Development
(d) Neither 1 nor 2
Brief report:
Explanation:
1. It has been released by the World
Statement 1 is correct Bank.
In respect of matters on which both the 2. As per this report, India is the
Parliament and the state legislatures have highest recipient of remittances in
power of legislation (i.e., the subjects the world.
enumerated in the Concurrent List), the
Which of the statements given above
executive power rests with the states except
is/are correct?
when a Constitutional provision or a
parliamentary law specifically confers it on (a) 1 only
the Centre. Therefore, a law on a concurrent (b) 2 only
subject, though enacted by the Parliament, is (c) Both 1 and 2
to be executed by the states except when the (d) Neither 1 nor 2
Constitution or the Parliament has directed Explanation:
otherwise.
Both the statements are correct
Statement 2 is correct
World Bank has released its Migration and
Article 265 says that no tax shall be levied or Development Brief. India has retained its
collected except by authority of law. position as the world‘s top recipient of
Article 263: Provision with respect to an remittances i.e. amount of money sent back
Inter-state council. home by its nationals working abroad for the
Source:LaxmikanthVth edition, chapter 14, year 2018.
page 14.7 and P.M Bakshi( see article 265) India has retained its top spot on
remittances, over the last three years and
72. Consider the following statements: has registered a significant flow of
1. The Chairman and members of a remittances from $62.7 billion in 2016 to
state public service commission $65.3 billion 2017.
are appointed and removed by the Source: KSG Current Connect, April 2019,
Governor of the state. page no 74
2. The chairman and members of the 74. Consider the following statements
Joint State Public Service about the Neelakurinji plant species:
Commission are appointed by the 1. It is native to Shola forests in
President. Western Ghats.
Which of the statements given above 2. It is categorized as critically
is/are correct? endangered as per the IUCN red
(a) 1 only list.
(b) 2 only Which of the statements given above
(c) Both 1 and 2 is/are correct?
(d) Neither 1 nor 2 (a) 1 only (b) 2 only
Explanation: (c) Both 1 and 2 (d) Neither 1 nor 2
Statement 1 is incorrect Explanation:
The Chairman and members of a state public Statement 1 is correct
service commission, though appointed by the

[TEST- 4 EXPLANATION] [KSG: 011-45552607, 47640303, 0141-4052441, 0755-4077441, 09386337412] Page 25

Googlw it: pdf4exams


https://t.me/pdf4exams www.pdf4exams.in

KSG: MEMBER’S COPY


Neelakurinji is a tropical plant species, Books and printing press, Labour welfare
native to Shola forests in Western Ghats. The were already there in Concurrent List.
flowers of Neelakurinji are purple-blue in Source:LaxmikanthVth edition, chapter 14,
colour and blooms once in 12 years. page 14.3
Statement 2 is incorrect
It is classified as Endangered. Paliyan tribal 77. According to the constitution of India,
people of Tamil Nadu use this flower bloom Parliament can legislate on a matter in
as reference to calculate their age. the State list when two or more states
Source: KSG Current Connect, April 2019, pass resolution requesting the
page no 64 Parliament to do so. Which of the
following statements regarding it are
75. Mount Agung which recently witnessed correct?
volcanic eruptions is located in: 1. Such a law can be amended or
(a) Indonesia repeal by both the Parliament and
the legislatures of the concerned
(b) Philippines
states.
(c) Japan
2. The state legislature ceases to have
(d) Mexico the power to make a law with
Explanation: respect to that matter.
Mount Agung on Indonesia's resort island of 3. The resolution operates as
Bali has erupted again, spewing volcanic ash abdication of the power of the state
into the sky to a height of two kilometres. legislature with respect to that
Two other volcanoes popular with tourists on matter entirely in the hands of
the island of Java also continue to erupt. Parliament.
Mount Bromo in East Java and Mount Select the correct answer using the
Merapi in Yogyakarta have exclusion zones code given below:
in place.
(a) 1 and 2 only (b) 2 and 3 only
Source: Current Connect April 2019, page
(c) 3 only (d) 1 and 3 only
no 91
Explanation:
Statement 1 is incorrect
76. Which of the subjects given below were
transferred from the State List to A law so enacted applies only to those states
Concurrent list in 42nd Constitutional which have passed the resolutions. However,
amendment act 1976? any other state may adopt it afterwards by
passing a resolution to that effect in its
1. Administration of Justice
legislature. Such a law can be amended or
2. Books and printing press repealed only by the Parliament and not by
3. Labour welfare the legislatures of the concerned states.
4. Forests Statement 2 and 3 are correct
5. Education The state legislature ceases to have the
Select the correct answer using the power to make a law with respect to that
code given below: matter. The resolution operates as abdication
(a) 2, 3 and 4 only or surrender of the power of the state
(b) 1, 3 and 5 only legislature with respect to that matter and it
is placed entirely in the hands of Parliament
(c) 1, 4 and 5 only
which alone can then legislate with respect
(d) 1, 2, 4 and 5 only to it.
Explanation: Source:Laxmikanth, Vth edition, chapter 14,
Statement 1, 4 and 5 are correct page 14.5 &14.6
The 42ndAmendment Act of 1976 transferred
five subjects toConcurrent List from State 78. ParamShivay which was recently in
List, i.e., (a) education, (b) forests, (c) weights news is a:
and measures, (d)protection of wild animals
(a) Indigenously built
and birds, and (e) administration of
supercomputer
justice;constitution and organisation of all
courts except the Supreme Court and the (b) Anti-Tank missile system
high courts. (c) Solar powered airplane
Statement 2 and 3 are incorrect (d) None of the above

[TEST- 4 EXPLANATION] [KSG: 011-45552607, 47640303, 0141-4052441, 0755-4077441, 09386337412] Page 26

Googlw it: pdf4exams


https://t.me/pdf4exams www.pdf4exams.in

KSG: MEMBER’S COPY


Explanation: The President can direct the states to reserve
‗ParamShivay‘ is the supercomputer of 833 money bills and other financial bills passed
teraflop capacity built at the cost of Rs 32.5 by the state legislature for his consideration
crore under the National Super Computing during a financial emergency.
Mission. It was recently inaugurated at the Source:LaxmikanthVth edition, chapter 14,
Indian Institute of Technology (IIT), Banaras page 14.7
Hindu University (BHU) by the Prime
Minister of India. A postal stamp and postal 81. Garia Puja is a Puja done by the tribes
stamp album were also released on the of:
centenary year of the institute.
(a) Tripura
Source: Current Connect April 2019, page
(b) Madhya Pradesh
no 94
(c) Kerala
(d) Meghalaya
79. ―Lord Howe Island‖ which is a UNESCO
world heritage site is located near: Explanation:
(a) Australia Garia Puja is a Puja done by the tribes of the
Tripura state. This festival is celebrated as a
(b) USA
harvest festival by the ethnic tribes and
(c) Peru celebrations are done during March-April.
(d) South Africa The festival is celebrated in a traditional way
Explanation: by the people of Tripura and celebrated
Lord Howe Island is located at some 600 km throughout the state with a lot of splendor
offshore from Sydney, Austrailia. It is home and joy. The Puja is mainly conducted by
to the world‘s southern-most coral reef Jhumias. A bamboo pole is worshipped with
containing many diverse and rare species not flowers and garlands by the tribalswhich
found anywhere else on the earth and is a symbolizes the Lord Garia, the deity of
UNESCO World Heritage Site. livestock and wealth.
Source: Current Connect April 2019, page Source: KSG Current Connect, April 2019,
no 94 page no 90

80. Consider the following statements: 82. TheBold Kurukshetra 2019 recently
1. A bill imposing restriction on the heard in news is a joint military
freedom of trade and commerce exercise between:
can be introduced in state (a) India and Singapore
legislature without previous (b) India and Indonesia
sanction of the President. (c) India and Nepal
2. The President can direct the states (d) India and Australia
to reserve money bills and other Explanation:
financial bills passed by the state
The 12th edition of joint military exercise
legislature for his consideration
between India and Singapore, BOLD
whenever he requires.
KURUKSHETRA 2019 was held recently. It
Which of the statements given above was a four day long joint training exercise
is/are correct? which focused on developing interoperability
(a) 1 only and conduct of joint tactical operations in
(b) 2 only mechanised warfare.
(c) Both 1 and 2 Source: KSG Current Connect April 2019,
(d) Neither 1 nor 2 page no 88
Explanation:
Statement 1 is incorrect 83. Consider the following:
Bills on certain matters enumerated in the 1. The National Integration Council
State List can be introduced in the state 2. NITI Aayog
legislature only with the previous sanction of 3. The University Grants Commission
the president. (For example, the bills Which of the institutions given above
imposing restrictions on the freedom of trade is/are the Constitutional bodies?
and commerce).
(a) 1 only
Statement 2 is incorrect
(b) 2 and 3 only

[TEST- 4 EXPLANATION] [KSG: 011-45552607, 47640303, 0141-4052441, 0755-4077441, 09386337412] Page 27

Googlw it: pdf4exams


https://t.me/pdf4exams www.pdf4exams.in

KSG: MEMBER’S COPY


(c) 1, 2 and 3 2.
When a person renounces his
(d) None of the above Indian citizenship, every minor
Explanation: child of that person also loses
Indian citizenship.
None of the above are constitutional bodies.
Which of the statements given above
The non-constitutional advisory bodies
is/are correct?
include:
(a) 1 only
The Planning Commission (now NITI Aayog),
the National Development Council, the (b) 2 only
National Integration Council, the Central (c) Both 1 and 2
Council of Health, the Central Council of (d) Neither 1 nor 2
Local Government and Urban Development, Explanation:
the Zonal Councils the North- Eastern
Council, the Central Council of Indian Statement 1 is incorrect
Medicine, Central Council of Homoeopathy,
the Central Family Welfare Council, the The children of foreign diplomats cannot
Transport Development Council, the acquire citizenship of India by birth. The
University Grants Commission and so on.
citizenship act 1955 incorporates provision
Source:LaxmikanthVth edition, chapter 14,
of acquiring Indian citizenship by birth
page 14.13 and 14.14
however it‘s not valid on foreign diplomats
and aliens.
84. Consider the following statements
about the exercise sea vigil: Statement 2 is correct
1. It is India‘s first national level Any citizen of India of full age and capacity
Coastal Defence Exercise.
can make a declaration renouncing his
2. All the Coastal states were involved
Indian citizenship. Upon the registration of
in this exercise.
that declaration, that person ceases to be a
Which of the statements given above
is/are correct? citizen of India. However, if such a
(a) 1 only declaration is made during a war in which
(b) 2 only India is engaged, its registration shall be
(c) Both 1 and 2 withheld by the Central Government.
(d) Neither 1 nor 2 Further, when a person renounces his
Explanation: Indian citizenship, every minor child of that
Both the statements are correct person also loses Indian citizenship.
Recently Indian Navy conducted exercise Sea However, when such a child attains the age
Vigil which was a maiden national level of eighteen, he may resume Indian
Coastal Defence Exercise. citizenship
The exercise witnessed the simultaneous
activation of the coastal security apparatus Source: Laxmikant Chapter 6 Citizenship.
across the country involving maritime
stakeholders at the Centre and all the 13 86. Consider the following statements
Coastal States and Union Territories. The about the National Emergency:
aim of the exercise is to provide a realistic 1. It can be proclaimed only when the
assessment of strengths and weakness of security of India or a part of it is
maritime security and in turn national threatened by war.
security. 2. During its operation, the state
Source: KSG Current Connect, April 2019, executive is dismissed.
page no 85 3. There is no maximum period
prescribed for its operation.
85. Consider the following statements Which of the statements given above
regarding Citizenship act 1955 in is/are correct?
India: (a) 1 only
(b) 1 and 3 only
1. The children of foreign diplomats (c) 2 only
born in India can acquire Indian (d) 1, 2 and 3
citizenship by birth.
[TEST- 4 EXPLANATION] [KSG: 011-45552607, 47640303, 0141-4052441, 0755-4077441, 09386337412] Page 28

Googlw it: pdf4exams


https://t.me/pdf4exams www.pdf4exams.in

KSG: MEMBER’S COPY


Explanation: 88. Consider the following statements with
Statement 1 is correct respect to Writs
It can be proclaimed only when the security 1. District courts can also issue
of India or a part of it is threatened by war, writs, only if enabled by the state
external aggression or armed rebellion. legislature
Statement 2 is incorrect 2. Supreme court may also refuse to
During its operation, the state executive and exercise its writ jurisdiction
legislature continue to function and exercise Which of the statements given above
the powers assigned to them under the is/are correct?
Constitution. Its effect is that the Centre gets (a) 1 only
concurrent powers of administration and (b) 2 only
legislation in the state. (c) Both 1 and 2
Statement 3 is correct (d) Neither 1 nor 2
There is no maximum period prescribed for Explanation:
its operation. It can be continued indefinitely
Statement 1 is incorrect: The Parliament
with the approval of Parliament for every six
(under Article 32) can empower any other
months.
court to issue these writs. Since no such
Source:LaxmikanthVth edition, Chapter 16, provision has been made so far, only the
page no 16.13 Supreme Court and the high courts can
issue the writs and not any other court
87. Consider the following statements Statement 2 is incorrect: A remedy under
about the powers and functions of the Article 32 is in itself a Fundamental Right
Prime Minister of India: and hence, the Supreme Court may not
1. The President can appoint only refuse to exercise its writ jurisdiction. On the
those persons as ministers who other hand, a remedy under Article 226 is
are recommended by the Prime discretionary and hence, a high court may
Minister. refuse to exercise its writ jurisdiction.
2. His resignation leads to collapse of SOURCE: Laxmikant (Ch- Fundamental
the council of Ministers. Rights)
3. He is the chief spokesman of the
Union government. 89. Consider the following:
Which of the statements given above 1. Coal
is/are correct? 2. Crude Oil
(a) 1 only (b) 1 and 2 only 3. Refinery Products
(c) 2 and 3 only (d) All of the above 4. Natural Gas
Explanation: 5. Fertilizers
Statement 1 is correct Which of the items given above are
He recommends persons who can be categorised as the core industries in
appointed as ministers by the president. The India?
President can appoint only those persons as (a) 1, 2 and 3 only
ministers who are recommended by the
(b) 3, 4 and 5 only
Prime Minister.
(c) 1, 3 and 5 only
Statement 2 is correct
(d) All of the above
He can bring about the collapse of the co-
uncil of ministers by resigning from office. Explanation:
Statement 3 is correct The eight core industries are Coal, Crude Oil,
Refinery Products, Natural Gas, Fertilizers,
He is the chief spokesman of the Union
Steel, Cement and Electricity.
government.
Source: KSG Current Connect April 2019,
Source:LaxmikanthVth edition, Chapter no
page no 68
19, page no 19.3, 19.4 &19.5

90. The word ‗Sovereign‘ written in the


Preamble of the constitution signifies
1. India is neither a dependent nor
a dominion of any other nation.

[TEST- 4 EXPLANATION] [KSG: 011-45552607, 47640303, 0141-4052441, 0755-4077441, 09386337412] Page 29

Googlw it: pdf4exams


https://t.me/pdf4exams www.pdf4exams.in

KSG: MEMBER’S COPY


2. India is free to conduct both (b) 2 only
external and internal affairs. (c) 1 and 3 only
3. India‘s membership of the United (d) 1, 2 and 3
Nations Organisation (UNO) Explanation:
might constitute a limitation on Statement 1 is correct
her sovereignty. All executive actions of the Government of
4. India can either acquire a foreign India are formally taken in his name. He can
territory or cede a part of its make rules specifying the manner in which
territory in favour of a foreign the orders and other instruments made and
state. executed in his name shall be authenticated.
He can make rules for more convenient
Which of the statements given above
transaction of business of the Union
is/are correct?
government, and for allocation of the said
(a) 1 and 3 only business among the ministers.
(b) 2 and 4 only Statement 2 is incorrect
(c) 1, 2 and 4 only He can summon or prorogue the Parliament
and dissolve the LokSabha. He can also
(d) 1, 2, 3 and 4 summon a joint sitting of both the Houses of
Explanation: Parliament, which is presided over by the
Speaker of the LokSabha.
Statement 1, 2 and 4 are correct, while
Statement 3 is correct
statement 3 is incorrect:
Money bills can be introduced in the
The word ‗sovereign‘ implies that India is Parliament only with his prior
neither a dependency nor a dominion of any recommendation.
other nation, but an independent state.
He causes to be laid before the Parliament
There is no authority above it, and it is free
the annual financial statement (i.e., the
to conduct its own affairs (both internal and
Union Budget).
external). Though in 1949, India declared the
continuation of her full membership of the Source:LaxmikanthVth edition, Chapter 17,
Commonwealth of Nations and accepted the page no 17.10, 17.11, 17.12
British Crown as the head of the
Commonwealth, this extra-constitutional 92. India is in the process of procuring the
declaration does not affect India‘s Spike-LR Anti-Tank Missiles from:
sovereignty in any manner. Further, India‘s (a) Israel
membership of the United Nations (b) Russia
Organisation (UNO) also in no way
(c) USA
constitutes a limitation on her
sovereignty.Being a sovereign state, India (d) Japan
can either acquire a foreign territory or cede Explanation:
a part of its territory in favour of a foreign The Indian Army is in the process of
state. procuring Spike-LR Anti-Tank Missiles from
Israel and Igla-S Very Short-Range Air
Source: Laxmikant , Chapter 4
Defence Systems (VSHORAD) from Russia
through a set of new financial powers for
91. Consider the following statements emergency procurements sanctioned by the
about the President of India: Defence Ministry.
1. All executive actions of the Source: KSG Current Connect, April 2019,
Government of India are formally page no 86
taken in his name.
2. He summons and presides over the 93. On which of the following
joint sitting of both the Houses of circumstances does the President act
Parliament. on his/her discretion?
3. Money bills can be introduced in 1. When the Prime Minister in office
the Parliament only with his prior dies suddenly and there is no
recommendation. obvious successor.
Which of the statements given above 2. Promulgating or withdrawing an
is/are correct? ordinance.
(a) 1 only
[TEST- 4 EXPLANATION] [KSG: 011-45552607, 47640303, 0141-4052441, 0755-4077441, 09386337412] Page 30

Googlw it: pdf4exams


https://t.me/pdf4exams www.pdf4exams.in

KSG: MEMBER’S COPY


3. Dissolution of the LokSabha if the 95. Consider the following statements
council of ministers has lost its about the Western Disturbances:
majority. 1. It originates in the Indian Ocean.
Select the correct answer using the 2. It is associated with rainfall,
codes given below: snowfall and fog in northern India.
(a) 1 only Which of the statements given above
(b) 2 only is/are correct?
(c) 1 and 3 only (a) 1 only
(d) 1, 2 and 3 (b) 2 only
Explanation: (c) Both 1 and 2
Statement 1 is correct (d) Neither 1 nor 2
Appointment of Prime Minister when no Explanation:
party has a clear majority in the LokSabha Statement 1 is incorrect
or when the Prime Minister in office dies Western disturbance (WD) is an extra-
suddenly and there is no obvious successor. tropical storm which originates in the
Statement 2 is incorrect Mediterranean region. The disturbance
The President can also withdraw an travels from the ―western‖ to the eastern
ordinance at any time. However, his power of direction. Disturbance means an area of
ordinance-making is not a discretionary ―disturbed‖ or reduced air pressure.
power, and he can promulgate or withdraw Statement 2 is correct
an ordinance only on the advice of the A WD is associated with rainfall, snowfall
council of ministers headed by the prime and fog in northern India. Upon its arrival in
minister. Pakistan and northern India, clouds along
Statement 3 is correct with rain and snow also arrive. The moisture
The President can act on his discretion in which WDs carry with them comes from the
these circumstances also. Mediterranean Sea and/or from the Atlantic
 Dismissal of the council of ministers Ocean.
when it cannot prove the confidence Source:KSG Current Connect, April 2019,
of the LokSabha. page no 65
 Dissolution of the LokSabha if the
council of ministers has lost its 96. Global Talent Competitiveness Index
majority. has been released by:
Source:LaxmikanthVth edition, Chapter 17, (a) United Nations Conference on
page no 17.19, 17.24 Trade and Development
(b) World Bank
94. The total number of ministers, (c) International Monetary Fund
including the Prime Minister, in the (d) INSEAD business school
Council of Ministers shall not exceed Explanation:
15% of the total strength of the
INSEAD business school in partnership with
LokSabha. This provision was added by
the: Tata Communications and Adecco Group
released Global Talent Competitive Index
(a) 61st Constitutional amendment act (GTCI) 2019.
(b) 74th Constitutional amendment act It was launched for the first time in 2013
(c) 89th Constitutional amendment act and since then it has become an annual
(d) 91st Constitutional amendment benchmarking report that measures the
act ability of countries to compete for talent.
Explanation: Source: KSG Current Connect, April 2019,
The total number of ministers, including the page no 30
Prime Minister, in the Council of Ministers
shall not exceed 15% of the total strength of 97. Consider the following statements:
the LokSabha. This provision was added by 1. General Security of Military
the 91st Amendment Act of 2003. Information Agreement (GSOMIA)
Source:LaxmikanthVth edition, Chapter 20, allows the sharing of classified
page no 20.2 information from the American

[TEST- 4 EXPLANATION] [KSG: 011-45552607, 47640303, 0141-4052441, 0755-4077441, 09386337412] Page 31

Googlw it: pdf4exams


https://t.me/pdf4exams www.pdf4exams.in

KSG: MEMBER’S COPY


companies with the Indian private 98. Consider the following statements
arms companies. about the International Maritime
2. Logistics Exchange Memorandum Organisation:
of Agreement (LEMOA) allows the 1. It works under the aegis of the
militaries of both countries access United Nations.
to each other‘s facilities for 2. It is vested with the responsibility
supplies and repairs. for the prevention of atmospheric
3. Communications Compatibility pollution by ships.
and Security Agreement 3. Its worksare in consonance with
(COMCASA) allows India to the UN Sustainable Development
procure transfer specialised Goals.
equipment for encrypted 4. Recently it hasmandated the
communications for US origin national governments to introduce
military platforms. electronic information exchange
Which of the statements given above between ships and ports.
is/are correct? Which of the statements given above
(a) 1 only are correct?
(b) 2 and 3 only (a) 1 and 2 only
(c) 3 only (b) 1, 3 and 4 only
(d) 1, 2 and 3 (c) 3 and 4 only
Explanation: (d) 1, 2, 3 and 4
Statement 1 is incorrect Explanation:
General Security of Military Information All the statements are correct
Agreement (GSOMIA) allows the sharing of IMO – the International Maritime
classified information from the U.S. Organization – is the United Nations
government and American companies with specialized agency with responsibility for the
the Government of India and Defence Public safety and security of shipping and the
Sector Undertakings (DPSU) but not with prevention of marine and atmospheric
Indian private companies. pollution by ships. IMO's work supports the
India signed GSOMIA in 2002. UN SDGs.
Statement 2 is correct A new global rule mandated by the
Logistics Exchange Memorandum of International Maritime Organisation (IMO)
Agreement (LEMOA) allows the militaries of for national governments to introduce
both countries access to each other‘s electronic information exchange between
facilities for supplies and repairs for the ships and ports took effect.
purpose of refuelling and replenishment in Source:KSG Current Connect April 2019,
primarily four areas — port calls, joint Page no 39
exercises, training and humanitarian
assistance and disaster relief. India
99. Consider the following statements
concluded LEMOA in 2016.
about the EMISAT satellite:
Statement 3 is correct
1. It has been jointly developed by
Communications Compatibility and Security India and Israel.
Agreement (COMCASA) allows India to
2. It is an advanced electronic
procure transfer specialised equipment for
intelligence (ELINT) satellite.
encrypted communications for US origin
military platforms like the C-17, C-130 and Which of the statements given above
P8Is. COMCASA is an India-specific version is/are correct?
of the Communication and Information on (a) 1 only (b) 2 only
Security Memorandum of Agreement (c) Both 1 and 2 (d) Neither 1 nor 2
(CISMOA). India concluded COMCASA in Explanation:
2018 on the side-lines of inaugural 2+2 Statement 1 is incorrect
dialogue. It is valid for a period of 10 years.
EMISAT has been jointly developed by ISRO-
Source: KSG Current Connect, April 2019, DRDO.
page no 36, 37
Statement 2 is correct
EMISAT is an advanced electronic
intelligence (ELINT) satellite.

[TEST- 4 EXPLANATION] [KSG: 011-45552607, 47640303, 0141-4052441, 0755-4077441, 09386337412] Page 32

Googlw it: pdf4exams


https://t.me/pdf4exams www.pdf4exams.in

KSG: MEMBER’S COPY


It is meant for electromagnetic spectrum (b) 1 and 2 only
measurements. Space-based electronic (c) 2 and 3 only
intelligence or ELINT will add teeth to (d) 1, 2 and 3
situational awareness of the Armed Forces as
Explanation:
it will provide location and information of
hostile radars placed at the borders. It is Statement 1 is incorrect
modelled after a famous Israeli spy satellite The Olive Ridley turtles are the smallest and
called SARAL (Satellite with ARgos and most abundant of all sea turtles found in the
ALtik(a). world, inhabiting warm waters of the Pacific,
Source: KSG Current Connect, April 2019, Atlantic and Indian oceans.
page no 50 Statement 2 is correct
Though found in abundance, their numbers
100. Consider the following statements have been declining over the past few years,
about the Olive Ridley Turtles: and the species is recognized as Vulnerable
by the IUCN Red list.
1. It is endemic to Indian Ocean.
Statement 3 is correct
2. It is recognized as Vulnerable by
the IUCN Red list. The mass nesting of Olive Ridley sea turtles
is at Odisha‘sGahirmatha Marine Sanctuary
3. Gahirmatha Marine Sanctuary of
in Kendrapara district. The sanctuary is
Odisha is known for mass nesting
considered to be the world‘s largest rookery
of this species.
of sea turtles.
Which of the statements given above
Source: KSG Current Connect April 2019,
is/are correct?
page no 62, 63
(a) 1 only

[TEST- 4 EXPLANATION] [KSG: 011-45552607, 47640303, 0141-4052441, 0755-4077441, 09386337412] Page 33

Googlw it: pdf4exams

Вам также может понравиться